Antepartum

¡Supera tus tareas y exámenes ahora con Quizwiz!

A pregnant client is positive for the human immunodeficiency virus (HIV). The nurse educates the client and determines that there is a need for further teaching if the client makes which statement?

"I can hold and diaper my newborn baby."2"Breastfeeding my newborn will be the best option for my baby."3"It may be as long as 2 years before I will know if my baby is HIV positive."4"If I take the prescribed medications for HIV, it is possible that I may not transfer this disease to my newborn." 2 It is very important that the nurse assess that the client has correct knowledge regarding the transmission of HIV and needed precautions to prevent the transmission of HIV.

The nurse is preparing a 36-year-old gravida II, para I pregnant client for an amniocentesis. She is at 16 weeks of gestation. Which action would the nurse take before the procedure to ensure fetal safety?

Require that the client empty her bladder.2Teach the client the signs and symptoms of labor.3Test the ultrasound equipment to ensure proper functioning.4Prepare a local anesthetic to be used during the insertion of the spinal needle. 3 Before 20 weeks of gestation, it is recommended to perform an amniocentesis with the bladder full. This pushes the uterus upward for better visualization. After week 20, the bladder is emptied before the test to minimize the risk of puncturing it during the test. Teaching the client about signs and symptoms of labor before the procedure does not ensure fetal safety. The local anesthetic makes the insertion of the needle less painful but does not protect the fetus. The use of ultrasound to guide the procedure has greatly decreased the risk of fetal and placental damage during the procedure.

The nurse is gathering data from a prenatal client with heart disease. The nurse carefully evaluates vital signs, monitors for weight gain, and checks the fluid and nutritional status. For which complication is the nurse collecting data?

Rh incompatibility2Fetal cardiomegaly3Increase in circulating volume4Hypertrophy and increased contractility 3 Pregnancy taxes the circulating system of every woman because both the blood volume and cardiac output increase. This is especially important to monitor in the client whose heart may not tolerate this normal increase. Hypertrophy may result in cardiac disease, but the outcome would be a decrease, not an increase, in contractility. Rh incompatibility and fetal cardiomegaly are related to the fetus, not the prenatal client.

The nurse is caring for a client diagnosed with preeclampsia. Which statement by the client suggests the need for further teaching regarding possible complications of preeclampsia?

"Blurred vision is not a normal occurrence."2"I will report any appearance of facial edema."3"I will be alert to any change in fetal movements."4"I should expect that my urine output will decrease." 4 Warning signs and symptoms of progression of preeclampsia to be reported include decreased urinary output; headaches and blurred vision; abdominal pain; and a change in fetal movement, particularly a decrease. Facial edema should be reported because the preeclampsia could be worsening.

A client calls the primary health care provider's office to schedule an appointment because a home pregnancy test was performed and the results were positive. The nurse determines that the home pregnancy test identified the presence of which in the urine?

Estrogen2Progesterone3Follicle-stimulating hormone4Human chorionic gonadotropin (hCG) 4 In early pregnancy, hCG is produced by trophoblastic cells that surround the developing embryo. This hormone is responsible for positive pregnancy tests. Options 1, 2, and 3 are incorrect.

The nurse is describing the process of fetal circulation to a client during a prenatal visit. The nurse would tell the client that fetal circulation consists of which components?

Two umbilical veins and one umbilical artery2Two umbilical arteries and one umbilical vein3Arteries that carry oxygenated blood to the fetus4Veins that carry deoxygenated blood to the fetus 2 Blood pumped by the fetus' heart leaves the fetus through two umbilical arteries. After the blood is oxygenated, it is then returned by one umbilical vein. The umbilical arteries carry deoxygenated blood and waste products from the fetus, and the umbilical vein carries oxygenated blood and provides oxygen and nutrients to the fetus.

The nurse is planning interventions for counseling a maternity client newly diagnosed with sickle cell anemia. The nurse understands that the important psychosocial intervention at this time is which action?

Provide emotional support.2Avoid the topic of the disease.3Allow the client to be alone if she is crying.4Provide all information regarding the disease immediately. 1 Provide emotional support.2Avoid the topic of the disease.3Allow the client to be alone if she is crying.4Provide all information regarding the disease immediately.

The nurse is reviewing the record of a client who has just been told that a pregnancy test is positive. Which probable signs of pregnancy refer to the softening of the uterus and related structures? Select all that apply.

Hegar's sign2Braxton Hicks3Goodell's sign4Chadwick's sign5McDonald's sign 1,3,5 Hegar's sign is a softening of the lower uterine segment. Because of the softening, it is easy to flex the body of the uterus against the cervix which is known as McDonald's sign. Goodell's sign is the softening of the cervix and the vagina caused by increased vascular congestion. Braxton Hicks contractions are irregular, painless uterine contractions that begin in the second trimester. Chadwick's sign is the purplish or bluish discoloration of the cervix, vagina, and vulva caused by increased vascular congestion.

During a prenatal visit of a client diagnosed with placenta previa, the primary health care provider defers doing a vaginal examination. The nurse understands that this examination is avoided in this situation because of what potential risk?

Initiating premature labor2Initiating severe hemorrhage3Causing rupture of the fetal membranes4Increasing the chance of uterine infection 2 The placenta is implanted low in the uterus with placenta previa, and cervical examination could cause the disruption of the placenta and initiate severe hemorrhage. While the other options are also correct, the greatest concern based on the information provided in the question is hemorrhage.

A prenatal client has acquired the sexually transmitted infection condyloma acuminatum (human papillomavirus). When assisting in planning care, which treatment would the nurse consider to be safe for this client?

Laser therapy2Use of cytotoxic agents3Treatment with interferon4All treatment should be avoided. 1 Laser therapy is the most effective method of treatment for this disorder that is considered safe for pregnancy. Medications for the disease are considered toxic to the fetus. The primary neonatal effect of the virus is respiratory or laryngeal papillomatosis. The exact route of perinatal transmission is unknown.

A nursing student enrolled in a physical assessment course is asked to describe the probable signs of pregnancy. The student displays correct understanding if the student lists which signs? Select all that apply.

Moro's2Hegar's3Chadwick's4McBurney's5McDonald's 2,3,5 Hegar's sign is softening of the lower uterine segment. This allows the body of the uterus to flex against the cervix, which is termed McDonald's sign. Chadwick's sign is a purple or blue discoloration of the cervix, vagina, and vulva caused by increased vascular congestion. Moro's sign is also called the startle reflex seen in normal newborns. McBurney's sign is pain in the lower right abdominal quadrant and is frequently seen in appendicitis.

The nurse is caring for a pregnant client with a history of human immunodeficiency virus (HIV). Which problem has the highest priority for this client?

Potential for infection2Inability to tolerate activity3Inability to maintain adequate nutritional intake4Inability to perform hygiene measures independently 1 Clients with HIV often show some evidence of immune dysfunction and may have increased vulnerability to infection. Although the client may have difficulty tolerating activity and need assistance with hygiene measures, these are not the priority concerns. Although imbalanced nutrition is a concern, infection is specifically related to HIV and is a priority.

The nurse in the prenatal clinic is collecting data regarding the client's nutritional knowledge. The nurse determines that the client understands the food items that are high in folic acid when the client states that she will be sure to eat which food items? Select all that apply.

Rice2Liver3Beans4Cheese5Chicken 2,3 Sources of folic acid include green, leafy vegetables; whole grains; fruits; liver; dried peas; and beans. Cheese is high in calcium, and rice and chicken are good sources of iron.

During a routine prenatal visit the client states, "I have not been able to get my wedding ring off for the past 2 days. I guess the heat is making my fingers swell." Which would the nurse check further?

The blood glucose level2The presence of vaginal discharge3Blood pressure changes and the presence of protein in the urine4Height of the fundus as compared with the date of the client's last visit 3 Finger edema is a frequent forerunner of gestational hypertension and should be investigated further. Blood glucose level, vaginal discharge, and height of the fundus are indicators of other problems such as diabetes, infection, and molar pregnancy, respectively.

A pregnant client is anxious to know the gender of the fetus and asks the nurse when she will be able to know. The nurse responds by telling the client that the gender of the fetus can usually be determined by which range of weeks?

6 to 8 2. 8 to 10 3. 12 to 16 4. 20 to 22 3 By the end of the twelfth week, the fetal gender can be determined by the appearance of the external genitalia on ultrasound.

The nurse is caring for a client receiving magnesium sulfate for preeclampsia. During the administration of this medication, which would the nurse specifically monitor?

Apical heart rate2Degree of edema3Deep tendon reflexes4Presence of pitting peripheral edema 3 Loss of reflexes is often the first sign of developing toxicity. The nurse should assess knee jerk (patellar tendon reflex) for evidence of diminished or absent reflexes. Although apical heart rate, degree of edema, and presence of pitting peripheral edema may be components of the assessment, these are not specifically associated with this medication.

The nurse is collecting data on a pregnant client in her 22nd week. The nurse prepares to use a fetoscope to auscultate the fetal heart rate. The nurse hears a fetal heart rate of 115 beats per minute. Which action would the nurse take?

Document the assessment.2Notify the primary health care provider.3Have another nurse check the fetal heart rate.4Compare the mother's pulse rate with the fetal heart rate. 1 Fetal heart sounds can be heard with a fetoscope by 18 to 20 weeks of gestation. The fetal heart rate ranges between a low of 110 to120 beats/minute and a high of 150 to 160 beats/minute. Since the fetal heart rate is normal, the nurse should document the findings. Notifying the primary health care provider, having another nurse recheck the assessment, and comparing maternal pulse rate to fetal pulse rate are unnecessary at this time.

The nurse is reinforcing a teaching session to a group of adolescent pregnant clients and is discussing the importance of nutrition. The nurse includes which information in the discussion?

Emphasizing the need to eliminate snack foods2Encouraging the need to avoid eating at local fast-food restaurants3Encouraging the adolescents to eat when hungry rather than three times a day4Describing the appropriate amount of weight gain required during the pregnancy 4 The developmental stage of the adolescent needs to be addressed when the nurse is providing instructions regarding nutrition during pregnancy. The adolescent should not be told to eliminate favorite foods and places to eat. This may cause the adolescent to rebel. Eating only when hungry could lead to a deficit in nutrients. The adolescent is more likely to follow suggestions when the nurse explains why the weight gain is important.

A client who is pregnant has been instructed on prevention of genital tract infections. Which statement by the client indicates an understanding of these prevention measures?

"I can douche any time I want."2"I can wear my tight-fitting jeans."3"I should avoid the use of condoms."4"I should choose underwear with a cotton panel liner." 4 Wearing items with a cotton panel liner allows for air movement in and around the genital area. Douching is to be avoided. Wearing tight clothes irritates the genital area and does not allow for air circulation. Condoms should be used to minimize the spread of sexually transmitted infectious diseases.

The nurse employed in a primary health care provider's office is collecting information from a pregnant client. Which statement made by the client indicates the need for psychological referral?

"I have terrible mood swings. I will be glad when this is all over."2"I will never be able to lose my weight and regain a great figure. I feel ugly."3"I don't like the way I look. My husband could never find me attractive again."4"I hate the way I look and feel. The baby has done this to me and I wish I were not pregnant." 4 Options 1, 2, and 3 are feelings about self and body image that many women express during pregnancy. The statement in option 4 is much stronger and places blame on the fetus. The direction of anger to the fetus should be explored. The nurse may find that a psychological referral is appropriate.

A perinatal client with a history of heart disease has been instructed on care at home. Which statement made by the client indicates the need for further teaching?

"I need to watch for weight gain."2"It is best to rest on my right side."3"I should avoid stressful situations."4"I need to avoid people with infections." 2 It is best to rest on the left side to promote blood return.

A maternity nurse is caring for a client who is admitted to the hospital with a diagnosis of gestational diabetes. This is the client's first pregnancy. Which statement by the client indicates a knowledge deficit regarding gestational diabetes?

"Well, I guess I will just have to deal with this."2"Oh, well, I guess this isn't the end of the world."3"I shouldn't have eaten so many sweets before I became pregnant."4"I have heard that this type of diabetes is first discovered during pregnancy." 3 Gestational diabetes is not necessarily caused by eating too many sweets before pregnancy. Options 1 and 2 indicate a common normal response. Option 4 is an accurate statement. Option 3 is the only option that indicates a knowledge deficit.

A nulliparous woman asks the nurse when she will feel fetal movements. The nurse responds by telling the woman that the first recognition of fetal movement will occur at approximately which week of gestation?

1. 10 2. 12 3. 14 4. 18 4 The first recognition of fetal movements, or "feeling life," by the multiparous woman may occur as early as the fourteenth to sixteenth week of gestation. The nulliparous woman may not notice these sensations until the 18th week of gestation or later. The first recognition of fetal movement is called quickening.

The nurse is providing information to a pregnant woman about food items high in folic acid. Which mid-afternoon snack should be recommended to supply folic acid?

1½ cups of yogurt2One medium banana3Nuts and green, leafy vegetables41 cup milk with two graham crackers 3 Folic acid is needed during pregnancy for healthy cell growth and repair. A pregnant woman should have at least four daily servings of foods rich in folic acid. Nuts and green, leafy vegetables contain folic acid. Milk and yogurt supply calcium. Bananas provide potassium.

The nurse is collecting data from a pregnant client who is currently at 28 weeks' gestation. At her prior prenatal visit, her fundal height measured 22 cm. The nurse measures the fundal height at this visit in centimeters and would expect which finding?

22 cm 2. 26 cm 3. 32 cm 4. 40 cm 2 During the second and third trimesters (18-30 weeks' gestation), the fundal height in centimeters approximately equals the fetus' age in weeks plus or minus 2 cm. In addition, at this point in the pregnancy, in a 4-week period, the fundal height should increase approximately 4 cm. At 14 to 16 weeks' gestation, the fundus can be located halfway between the symphysis pubis and the umbilicus. At 20 to 22 weeks' gestation, the fundus is at the umbilicus, and at term, the fundus is at the xiphoid process.

A primigravida client comes to the clinic and has been diagnosed with a urinary tract infection. She has repeatedly verbalized concern regarding safety of the fetus. Which client problem does the nurse identify as the priority at this time?

Altered tissue integrity2Urinary tract infection3Pain associated with the infection4Fear about the well-being of the fetus 4 The primary concern for this client is safety of her fetus, not herself. Therefore, the priority problem at this time is fetal well-being. Urinary tract infection is the client's diagnosis. Altered tissue integrity and pain may be seen in clients experiencing urinary tract infections, but the question includes no data to support either of the options

A client presents at her primary health care provider's office 10 weeks pregnant with her first pregnancy. Which are presumptive signs of pregnancy that the client might be expected to have? Select all that apply.

Fatigue2Breast changes3Chadwick's sign4Nausea and vomiting5Pigmentation changes of the face6A bluish discoloration of the vagina and cervix 1,2,4 Fatigue, breast changes, nausea and vomiting, and amenorrhea are presumptive signs of pregnancy. Chadwick's sign, a bluish discoloration of the vagina and cervix, is a probable sign. Pigmentation changes of the face are not a sign of pregnancy, although they may occur with pregnancy.

The nurse is preparing to monitor a fetal heart rate. The nurse locates a round, ballottable shape just above the symphysis pubis. Fetal small parts are located on the right side of the uterus with a concave shape located on the left side of the uterus. Where would the nurse listen to hear the strongest fetal heart tones?

Left lower quadrant2Left upper quadrant3Right lower quadrant4Right upper quadrant 1 The fetal heart rate is best detected through the fetal back. The findings in this situation support a cephalic presentation. The extremities are on the right side, and the back is on the left side. The fetal heart rate is best heard in the left lower quadrant.

The nurse is instructing a pregnant client on dietary sources of iron. Which client food selection demonstrates an understanding of teaching?

Milk2Potatoes3Cantaloupe4Fresh spinach 4 Dietary sources of iron include lean meats; liver; shellfish; dark green, leafy vegetables such as spinach; legumes; whole grains and enriched grains; cereals; and molasses. Milk is high in calcium and also contains phosphorus. Potatoes and cantaloupe are high in vitamin C.

A pregnant woman who is at 38 weeks' gestation arrives at the emergency department. She reports the presence of bright red vaginal bleeding and denies the presence of any pain. Based on this information, what does the nurse determine the client may be experiencing?

Placenta previa2Abruptio placentae3Rupture of the amniotic sac4The passage of the mucous plug 1 The primary sign in placenta previa is painless vaginal bleeding in the second or third trimester of pregnancy. Findings of abruptio placentae include dark red vaginal bleeding and abdominal pain. A ruptured amniotic sac would include findings such as a watery vaginal drainage. Passage of the mucous plug appears pink or as blood-tinged mucus.

The nurse is collecting data from a client who is pregnant with twins. The nurse understands that which complications are more likely to occur with a twin pregnancy? Select all that apply.

Preterm labor2Postterm labor3Maternal anemia4Oligohydramnios5Gestational diabetes 1,3 There is a higher incidence of preterm labor occurring in multiple gestations. In addition, maternal anemia occurs in a client pregnant with twins because the maternal system is nurturing more than one fetus. Preterm labor, rather than postterm labor is likely to occur. Hydramnios not oligohydramnios may be associated with a twin pregnancy as a result of increased renal perfusion from cross-vessel anastomosis with monozygotic twins. Gestational diabetes is not a complication of a twin pregnancy.

The nurse is teaching a pregnant woman about the physiological effects and hormone changes that occur in pregnancy, and the woman asks the nurse about the purpose of progesterone. According to the nurse, what is the purpose of progesterone?

Progesterone maintains the uterine lining for implantation.2Progesterone stimulates metabolism of glucose and converts the glucose to fat.3Progesterone prevents the involution of the corpus luteum and maintains the production of progesterone until the placenta is formed.4Progesterone stimulates uterine development to provide an environment for the fetus, and stimulates the breasts to prepare for lactation. 1 Progesterone maintains the uterine lining for implantation and relaxes smooth muscle. Human placental lactogen stimulates the metabolism of glucose and converts the glucose to fat and is antagonistic to insulin. Human chorionic gonadotropin prevents involution of the corpus luteum and maintains the production of progesterone until the placenta is formed. Estrogen stimulates uterine development to provide an environment for the fetus and stimulates the breasts to prepare for lactation.

A pregnant client is receiving magnesium sulfate for the management of preeclampsia. The nurse determines that the client is experiencing toxicity from the medication if which findings are noted during assessment? Select all that apply

Proteinuria of 3+2Respirations of 10 breaths/minute3Presence of deep tendon reflexes4Urine output of 20 mL in an hour5Serum magnesium level of 6 mEq/L (3 mmol/L) 2,4 Magnesium toxicity can occur from magnesium sulfate therapy. Signs of magnesium sulfate toxicity relate to the central nervous system depressant effects of the medication and include respiratory depression, loss of deep tendon reflexes, and a sudden decline in fetal heart rate and maternal heart rate and blood pressure. Urine output should be at least 25 mL to 30 mL per hour. Therapeutic serum levels of magnesium are 4 mEq/L to 7 mEq/L (2 to 3.5 mmol/L). Proteinuria of 3+ is an expected finding in a client with preeclampsia.

The nurse is monitoring a pregnant client with gestational hypertension (GH) who is at risk for preeclampsia. The nurse would check the client for which signs of preeclampsia? Select all that apply.

Proteinuria2Hypertension3Low-grade fever4Increased pulse rate5Increased respiratory rate 1,2 Signs of preeclampsia are hypertension and proteinuria. A low-grade fever, increased pulse rate, and increased respiratory rate are not associated with preeclampsia.

The nurse caring for a client with abruptio placentae is monitoring the client for signs of disseminated intravascular coagulopathy (DIC). The nurse would suspect DIC if which is observed?

Rapid clotting times2Pain and swelling of the calf of one leg3Laboratory values that indicate increased platelets4Petechiae, oozing from injection sites, and hematuria 4 DIC is a state of diffuse clotting in which clotting factors are consumed, which leads to widespread bleeding. Platelet counts are decreased, because they are consumed by the process. Coagulation studies show no clot formation (clotting times are thus prolonged), and fibrin plugs may clog the microvasculature diffusely rather than in an isolated area.

The nurse is teaching a pregnant client how to perform Kegel exercises. The nurse would tell the client that these exercises are for which purpose?

Reduce a backache.2Prevent ankle edema.3Prevent urinary tract infections.4Strengthen the pelvic floor in preparation for delivery. 4 Kegel exercises will assist in strengthening the pelvic floor. Pelvic tilt exercises will help reduce backaches. Leg elevation will assist in preventing ankle edema. Instructing a client to drink 8 ounces of fluids 6 times a day will help prevent urinary tract infections.

The nurse instructs a pregnant client diagnosed with human immunodeficiency virus (HIV) to report immediately to the primary health care provider any early signs of vaginal discharge or perineal tenderness. Which is the primary expected outcome for this intervention?

Relieves anxiety for the pregnant client2Eliminates the need for further unnecessary screenings3Assists in identifying infections that may need to be treated4Minimizes the financial cost of caring for an HIV-positive client 3 The HIV-positive client may be further at risk for superimposed infections during pregnancy. Among these include Candida infections, genital herpes, and anogenital condyloma. Early reporting of symptoms may alert the members of the health care team that further assessment and testing are needed to diagnose and manage additional maternal and fetal physiological risks. The remaining options are benefits that can be experienced when complications such as infections are identified early.

The nurse is assigned to care for a client admitted with severe preeclampsia. Which is the priority nursing intervention for this client?

Restricting food and fluids2Monitoring blood glucose levels3Maintaining the client in a supine position4Minimizing the client's exposure to external stimuli 4 The client with severe preeclampsia is kept on complete bed rest in a quiet environment. External stimuli such as lights, noise, and visitors that may precipitate a seizure should be kept to a minimum. Food and fluid are not restricted unless prescribed by the care provider. There is no relationship between severe preeclampsia and blood glucose levels. The client is instructed to rest in a left lateral position to decrease pressure on the vena cava, thereby increasing cardiac perfusion of vital organs.

A pregnant client asks the nurse about the type of exercises that are allowable during the pregnancy. The nurse would instruct the client that which is the safest exercise?

Swimming2Scuba diving3Low-weight gymnastics4Bicycling with the legs in the air 1 Non-weight-bearing exercises are preferable to weight-bearing exercises. Competitive or high-risk sports such as scuba diving, water skiing, downhill skiing, horseback riding, basketball, volleyball, and gymnastics should be avoided. Non-weight-bearing exercise such as swimming is allowable. Exercises to avoid are shoulder standing and bicycling with the legs in the air because the use of the knee-chest position should be avoided.

During an office visit, a prenatal client with mitral stenosis states she has been under a lot of stress lately. During data collection, the client questions everything the nurse does and behaves in an anxious manner. Which is the appropriate nursing response or action at this time?

Tell the client not to worry.2Refer the client to a counselor.3Ignore the client's unfounded concerns and continue.4Explain the purpose of the nurse's actions and answer all questions. 4 For the prenatal cardiac client, stress should be reduced as much as possible. It is important to be certain the woman understands the purpose of any procedures so she does not worry unnecessarily. Options 1, 2, and 3 are nontherapeutic at this time. Explaining the purpose of nursing actions will assist in decreasing the stress level of the client.

The nurse is preparing a pregnant client for a transvaginal ultrasound exam. The nurse should tell the client that which will occur?

The client will be placed in the supine position.2The client will feel some pain during the procedure.3The client will feel some pressure when the vaginal probe is moved.4The client will need to drink 2 quarts of water to attain a full bladder. 3 Transvaginal ultrasonography, in which a lubricated probe is inserted into the vagina, allows evaluation of the pelvic anatomy. A transvaginal ultrasound exam is well tolerated by most clients because it alleviates the need for a full bladder. The client is placed in a lithotomy position or with her pelvis elevated by towels, cushions, or a folded blanket. The procedure is not physically painful, although the woman may feel pressure as the probe is moved.

The nurse is performing an assessment on a pregnant client who has had a severe asthma attack. The nurse asks the client about prescription and herbal medications she is taking, and the client tells the nurse that she has been taking the herb chamomile. Which statement made by the client demonstrates correct information about this herbal intervention?

"Chamomile is a known stimulant."2"Chamomile is always safe for women and children."3"Chamomile has no side effects or interactions with other medications."4"Chamomile should not be used while I am pregnant and because I have asthma." 4 Chamomile is a known abortifacient and relaxant and should not be used during pregnancy. Persons with asthma should not use this herb because cross-hypersensitivity may result in persons who are allergic to sunflowers, ragweed, or members of the aster family. Options 1, 2, and 3 are incorrect statements regarding chamomile.

The nurse reinforces instructions to a client with mild preeclampsia on home care. Which comment by the client indicates that teaching is effective?

"I need to check my weight every day at different times during the day."2"I need to take my blood pressure each morning and alternate arms each time."3"As long as the health nurse is visiting me daily, I do not have to keep my next primary health care provider's appointment."4"I need to check my urine with a dipstick every day for protein and call my health care provider if it is 2+ or more." 4 The client diagnosed with mild preeclampsia may be treated in the home setting. Option 4 is a correct statement. The weight must be checked at the same time each day under the following conditions to obtain reliable weights: client wearing the same clothes, after client voids, and before client eats breakfast. Blood pressure must be taken in the same arm, in a sitting position, every day to obtain a consistent and accurate reading. It is still important to keep primary health care provider appointments to monitor for any other physical changes in the mother or baby.

A client is pregnant, has a history of heart disease, and has been instructed on care at home. Which statement by the client indicates that she understands her needs?

"I should rest on my back."2"My weight gain is not important."3"I should avoid stressful situations."4"There is no restriction on people who visit me." 3 To avoid infections, visitors with active infections should not be allowed to visit the client. Stress causes increased heart workload. Too much weight gain causes an increase in body requirements and stress on the heart. Resting should be on the side to prevent vena cava syndrome (hypotensive syndrome) and to promote blood return.

A pregnant anemic client is concerned about her baby's condition following delivery. Which nursing response best supports the client?

"I wouldn't worry about your baby's health; complications from this condition are generally rare."2"Your baby will likely need to spend a few days in the neonatal intensive care unit for observation following delivery."3"Your baby will not have any problems if you follow all the advice the primary health care provider has given you during your pregnancy."4"The effects of anemia on your baby are difficult to predict, but let's review your plan of care to ensure you are providing the best nutrition and growth potential." 4 The effects of maternal iron deficiency anemia on the developing fetus and neonate are unclear. In general, it is believed that the fetus will receive adequate maternal stores of iron, even if a deficiency is present. Neonates of severely anemic mothers have been reported to experience reduced red cell volume, hemoglobin, and iron stores. Option 4 provides the most realistic support for the client and allows the nurse an opportunity to review the client's plan of care to clarify information and reassure the mother.

A client who is in the second trimester of pregnancy develops melasma during pregnancy. Which statements made by the client indicate an understanding of this condition? Select all that apply

"Melasma may reoccur in a subsequent pregnancy."2"My stretch marks will turn less noticeable after I have the baby."3"This dark line down my abdomen will fade at the end of my pregnancy."4"These brown, splotchy patches will most likely disappear after I deliver my baby."5"The dark patches that are on my nose, cheeks, and forehead will most likely darken until the baby is delivered." 1,4,5 Melasma, also known as chloasma, affects 50% to 70% of all pregnancies after the 16th week of gestation. Once melasma has occurred in the pregnant woman, the chances of it occurring in a future pregnancy are increased. The characteristic splotches of melasma tend to disappear following the end of the pregnancy. These splotches become progressively darker the closer the mother gets to term.

A client at 32 weeks of gestation with a diagnosis of severe preeclampsia is admitted to the maternity department. The client is alone and appears very anxious. Which statement by the nurse is therapeutic?

"Tell me about your concerns."2"Your husband called to say he's coming to be with you."3"Many women have this problem with no further complications."4"You have an excellent primary health care provider; if anyone can save your baby, she can." 1 The client is apprehensive and the nurse needs to assist the client in exploring her feelings and concerns. The remaining options do not focus on the client's feelings. Additionally, there are no data to suggest the client is married.

A pregnant client is seen in the health care clinic and asks the nurse what causes the breasts to change in size and appearance during pregnancy. Which response is appropriate for the nurse to make?

"The breasts become stretched because of weight gain."2"The increased metabolic rate causes the breasts to become larger."3"The breast changes are a result of the secretion of estrogen and progesterone."4"Cortisol secreted by the adrenals plays a factor in increasing the size and appearance of the breasts." 3 During pregnancy the breasts change in both size and appearance. The increase in size is a result of the effects of estrogen and progesterone on maternal tissue and structures. Estrogen stimulates the growth of mammary ductal tissue, and progesterone promotes the growth of lobes, lobules, and alveoli. A delicate network of veins is often visible just beneath the surface of the skin. This is the most direct, comprehensive response.

A pregnant woman reports that she has just finished taking the prescribed antibiotics to treat a urinary tract infection. The mother expresses concern that her baby will be born with an infection. Which response would the nurse make to help reduce the maternal fears that the newborn will be born with an infection?

"Urinary infections during pregnancy are common. Your baby will be fine."2"Your developing baby cannot acquire an infection from you during pregnancy."3"You shouldn't worry about this because you received early prenatal care and are taking your prenatal vitamins."4"Now that you have taken the medication as prescribed, we will continue to monitor you closely by repeating the urine culture before you leave today." 4 Symptomatic bacteriuria has been associated with an increased risk of neonatal sepsis following delivery. Appropriate antenatal care of a client with a urinary tract infection includes antibiotic treatment and follow-up repeat urine cultures. Option 4 is the only response that is both therapeutic and identifies accurate information.

A client is 8 weeks pregnant and has waves of nausea accompanied by vomiting throughout the day. Food odors consistently precipitate the nausea. Her husband has an important business dinner planned, and she is reluctant to attend because of the nausea and vomiting. This has placed a strain on the husband-wife relationship. Which statement by the nurse indicates an understanding of the problem?

"You are afraid your husband will go to dinner without you."2"You feel you are having difficulty fulfilling your role as a wife."3"You are not physically able to go to dinner and should stay at home."4"You should go to dinner. Others will understand if you don't feel well." 2 By identifying the feeling that the client is experiencing, the nurse provides the opportunity for further discussion. Option 2 reflects a feeling that the woman may be experiencing. There are no data to support the fear that the wife will be left at home. Options 3 and 4 are comparable or alike as both are examples of giving advice and do not lead to open communication with the pregnant woman.

During a prenatal visit, the nurse checks the fetal heart rate (FHR) of a client in the third trimester of pregnancy. The nurse determines that the FHR is normal if which heart rate is noted?

80 beats per minute 2. 100 beats per minute 3. 150 beats per minute 4. 180 beats per minute 3 Fetal heart rate depends on gestational age. It is normally 160 to 170 beats per minute during the first trimester, but it slows with fetal growth to 110 to 160 beats per minute near or at term.

The clinic nurse is reviewing the records of the pregnant clients who will be seen in the clinic. Which client profile presents the greatest risk for human immunodeficiency virus (HIV) infection?

A 33-year-old gravida III2An adolescent with multiple heterosexual contacts3A multigravida with a history of repeat cesarean deliveries4A 25-year-old client with a history of spontaneous abortions 2 Although all women are at risk for developing HIV during their reproductive years, it is believed that adolescents are particularly at risk because they engage in high-risk behaviors. The client profiles in options 1, 3, and 4 identify at-risk situations for a variety of obstetric risk factors but not necessarily HIV infection.

When collecting data from a pregnant client at risk for disseminated intravascular coagulation (DIC), which factors would the nurse consider significant?

A client who is primigravida with mild preeclampsia2A client who is primigravida who delivered a 10-pound baby 3 hours ago3A client who is gravida VI who delivered 10 hours ago and has lost 450 mL of blood4A client who is gravida II who has just been diagnosed with dead fetus syndrome; fetal demise occurred 2 months ago 4 Dead fetus syndrome is considered a risk factor for DIC. Severe preeclampsia is considered a risk factor for DIC; a mild case is not. Delivering a large baby is not considered a risk factor for DIC. Hemorrhage is a risk factor with DIC; however, a loss of 450 mL is not considered hemorrhage.

The clinic nurse is preparing to discuss cardiovascular changes of pregnancy in a prenatal class. Which information is appropriate for the nurse to present to this group? Select all that apply.

A hemoglobin count that falls to 13 g/dL indicates anemia.2The number of red blood cells will be increased during pregnancy.3At term, the heart rate has increased by 15 to 20 beats per minute.4A reduction in coagulation factors protects against thrombus formation.5The white blood cell count will slowly decrease over the course of the pregnancy.6In a supine position, some degree of compression of the vena cava will occur. 2,3,6 Red blood cell production is increased during pregnancy and supine hypotension is a risk for the pregnant woman. The heart rate increases by 15 to 20 beats per minute. The hemoglobin count needs to decrease to below 11 g/dL to indicate anemia. Coagulation factors increase during pregnancy and place the pregnant woman at an increased risk for deep vein thrombosis. The white blood cell count increases slightly over the course of the pregnancy.

The perinatal client is admitted to the obstetrical unit during an exacerbation of a heart condition. When planning for the nutritional requirements of the client, the nurse would consult with the dietitian to ensure which dietary measure?

A low-calorie diet to ensure the absence of weight gain2A diet that is high in fluids and fiber to decrease constipation3A diet that is low in fluids and fiber to decrease blood volume4Unlimited sodium intake to increase the circulating blood volume 2 Constipation causes the client to use Valsalva's maneuver. This causes blood to rush to the heart and overload the cardiac system. The absence of weight gain is not recommended during pregnancy. Diets that are low in fluid and fiber cause a decrease in blood volume, which in turn deprives the fetus of nutrients. Too much sodium could cause an overload to the circulating blood volume and contribute to the cardiac condition.

A nonstress test is performed on a client, and the results are documented in the chart. The results are documented as a reactive nonstress test. Which interpretation would the nurse make of these results?

A positive test2A negative test3A suspicious test4An unsatisfactory test 2 A reactive nonstress test (normal/negative) indicates a healthy fetus. A nonreactive nonstress test is an abnormal test and requires further follow-up. A suspicious test result also requires further follow-up. An unsatisfactory test cannot be interpreted because of the poor quality of the fetal heart rate findings.

The nurse is reviewing the record of a client who has just been told that her pregnancy test is positive. The nurse notes that the primary health care provider has documented the presence of Goodell's sign. The nurse determines that this sign is indicative of which change that occurs with pregnancy?

A softening of the cervix2The presence of fetal movement3The presence of human chorionic gonadotropin in the urine4A soft blowing sound that corresponds with the maternal pulse that is heard while auscultating the uterus 1 During the early weeks of pregnancy, the cervix becomes softer as a result of pelvic vasoconstriction, which causes Goodell's sign. Cervical softening is noted by the examiner during a pelvic examination. Goodell's sign does not indicate the presence of fetal movement. Human chorionic gonadotropin is noted in maternal urine with a positive urine pregnancy test. A soft blowing sound that corresponds with the maternal pulse may be auscultated over the uterus; it is the result of blood circulating through the placenta.

The nurse is collecting data from a client and is reviewing the client's health record to determine the risk for preterm labor. Which finding places the client at risk for preterm labor?

A urinary tract infection2A single-fetus pregnancy3A 26-year-old primigravida4A hemoglobin of 13.5 g/dL 1 One risk factor for preterm labor is the presence of a genitourinary infection. Although the connection is not clearly understood, one hypothesis involves the release of prostaglandins by the pathogens, which may contribute to the initiation of contractions. Other risk factors for preterm labor include a multifetal pregnancy, which contributes to overdistention of the uterus; anemia, which decreases oxygen supply to the uterus; and age less than 15 years or first pregnancy older than the age of 35.

The nurse is collecting data on a pregnant woman who is diagnosed with human immunodeficiency virus (HIV) during the thirty-second gestational week. The nurse reviews the data and determines that which finding requires further follow-up?

Active fetal movement2Weight gain of 22 pounds3Slight lower extremity edema4Increased shortness of breath and bilateral rales 4 HIV infection in a pregnant woman may cause both maternal and fetal complications. Fetal compromise can occur because of premature rupture of the membranes, preterm birth, or low birth weight. Potential maternal effects include an increased risk of opportunistic infections. Individuals in the later stages of HIV are further susceptible to other invasive conditions, such as tuberculosis and a wide variety of bacterial infections. The findings of shortness of breath and bilateral rales can be indicative of an opportunistic infection and require follow-up.

207 of 233GO When planning care for a woman with gestational hypertension (GH), the nurse plans to encourage which maternal behavior?

Anticipatory grieving2Walking 1 to 2 miles daily3Expression of hope for a positive outcome4Delaying preparations for finishing the nursery at home 3 Hoping for a positive outcome is an appropriate coping mechanism. It is important to support an expression of hope by a client with a high-risk pregnancy as long as the hope is realistic (e.g., fetus is viable). Anticipatory grieving is not a positive adaptation for this client. Grieving should begin when a loss occurs. Walking 1 to 2 miles daily is contraindicated for a woman with gestational hypertension. Delaying nursery preparations at home reflects an "expecting the worst" situation.

A client asks the nurse to describe how her developing baby will get enough blood and oxygen. The nurse responds that the fetal circulatory system accomplishes this task by which means? Select all that apply.

Completely shutting off circulation to fetal lung tissue2Bypassing the fetal lungs to circulate oxygen rich blood3Using the fetal lungs and liver to promote gas exchange4Using the fetus's beating heart to pump blood in the circulatory system5Carrying more oxygen on fetal hemoglobin than maternal hemoglobin6Making the fetal cardiac output higher per unit of body weight than the maternal cardiac output 2,4,5,6 The fetal lungs do not function for respiratory gas exchange, so a special circulatory pathway, the ductus arteriosus, bypasses the lungs. A small amount of blood circulates through the resistant lung tissue, but the majority follows the path with less resistance through the ductus arteriosus into the aorta. The following three special characteristics enable the fetus to obtain sufficient oxygen from maternal blood: (1) fetal hemoglobin carries 20% to 30% more oxygen than maternal hemoglobin, (2) the hemoglobin concentration of the fetus is about 50% greater than that of the mother, and (3) the fetal heart rate is 110 to 160 beats per minute, making the cardiac output per unit of body weight higher than that of an adult.

A pregnant client is seen in the health care clinic for a regular prenatal visit. The client tells the nurse that she is experiencing irregular contractions, and the nurse determines that the client is experiencing Braxton Hicks contractions. Which nursing action would the nurse take?

Contact the primary health care provider.2Instruct the client to maintain bed rest for the remainder of the pregnancy.3Instruct the client that these are common and may occur throughout the pregnancy.4Call the maternity unit and inform them that the client will be admitted in a prelabor condition. 3 Braxton Hicks contractions are irregular, painless contractions that occur throughout pregnancy, although many expectant mothers do not notice them until the third trimester. Because Braxton Hicks contractions may occur and are normal in some pregnant women during pregnancy, contacting the primary health care provider, maintaining bed rest for the remainder of the pregnancy, and admitting the client to the labor and delivery unit are unnecessary and inaccurate.

A pregnant client is seen in the health care clinic for a regular prenatal visit. The client tells the nurse that she is experiencing irregular contractions, and the nurse determines that the client is experiencing Braxton Hicks contractions. Which nursing action would be appropriate?

Contact the primary health care provider.2Instruct the client to maintain bed rest for the remainder of the pregnancy.3Instruct the client that these are common and may occur throughout the pregnancy.4Call the maternity unit and inform them that the client will be admitted in a prelabor condition. 3 Braxton Hicks contractions are irregular, painless contractions that occur throughout pregnancy, although many expectant mothers do not notice them until the third trimester. Because Braxton Hicks contractions may occur and are normal in some women during pregnancy, contacting the primary health care provider, maintaining bed rest, and admitting the client to the labor unit are unnecessary and inaccurate interventions.

The nurse is teaching a prenatal class on the anatomy and physiology of the female reproductive system including hormones. Estrogen produces which effects, either directly or indirectly, during pregnancy? Select all that apply.

Decreases salivation2Decreases skin pigmentation3Increases blood flow to the uterine vessels4Stimulates development of the breast ducts5Causes vascular changes in the mucous membranes of the nose and mouth 3,4,5 Estrogen increases the blood flow to the uterine blood vessels. It stimulates the development of the breast ducts in preparation for lactation. It also increases vascular changes in the skin and the mucous membranes of the nose and mouth. It also increases, rather than decreases, salivation. Estrogen also increases skin pigmentation, which accounts for the "mask of pregnancy."

The nurse is reinforcing instructions to a pregnant client regarding measures that will strengthen the perineal floor muscles. Which would the nurse include in the instructions?

Drink 8 ounces of fluid 6 times per day.2Wipe the perineal area anterior to posterior after toileting.3Perform Kegel exercises in 10 repetitions, three times per day.4Perform pelvic tilt exercises in 10 repetitions, three times per day. 3 Kegel exercises strengthen the pelvic floor. Option 1 relates to hydration that is important for normal physiological body functioning. Option 2 will help prevent urinary tract infections. Pelvic tilt exercises will reduce backache.

A client who is 6 months pregnant is attending her first prenatal visit. On the first prenatal visit, the nurse notes that the client is gravida 4, para 0, abortion 3. The client is 5 feet, 6 inches tall, weighs 130 pounds, and is 25 years old. She states, "I get really tired after working all day and can't keep up with my housework." Which factor in the above data would lead the nurse to suspect gestational diabetes?

Fatigue2Obesity3Fetal demise4Maternal age 3 A previous history of unexplained stillbirths or miscarriages puts the client at high risk for gestational diabetes. Fatigue is a normal occurrence during pregnancy. A client at 5 feet, 6 inches tall, 130 pounds does not meet the criteria of 20% over ideal weight; therefore, the client is not obese. To be at high risk for gestational diabetes, the maternal age should be greater than 30 years.

The nurse is assisting in performing a prenatal examination on a client in the third trimester of pregnancy. The primary health care provider performs Leopold's maneuvers on the client. Which maneuver indicates the position of the fetus?

First2Second3Third4Fourth 2 The first maneuver is to determine the presentation and lie of the fetus. The second maneuver indicates the position of the fetus. The third maneuver can determine whether the fetus is engaged in the pelvis. The fourth maneuver indicates the attitude of the fetus. Leopold's maneuvers should not be performed during a contraction.

A woman is 24 weeks pregnant. She had a previous stillborn neonate at 38 weeks' gestation and a pregnancy that ended at 34 weeks with the birth of a stillborn girl. She states she has a 4-year-old son and an 8-year-old daughter who live with her at home and were both born at 38 weeks. What is her gravidity and parity, using the five-digit system (GTPAL)?

G (4) T (3) P (0) A (0) L (2) 2. G (4) T (1) P (2) A (0) L (2) 3. G (5) T (3) P (1) A (0) L (2) 4. G (5) T (0) P (4) A (0) L (2) 4 The woman has had 4 previous pregnancies and is now pregnant. This would indicate that she is a gravida (G) 5. She has had 3 term (T) pregnancies; the other 1 was classified as preterm (P) because it occurred between 20 and 37 weeks' gestation. She has had no miscarriages or abortions (A) leading this calculation to a 0. Finally, she has 2 living (L) children at home.

The nurse is caring for a prenatal client who is at risk for placental abruption. Which risk factor documented in the client's record supports this risk factor?

Gestational diabetes2Maternal hypertension3Hyperemesis gravidarum4Previous cesarean section 4 It is possible that placental abruption can result from maternal hypertension, which causes degenerative changes in the small arteries that supply intervillous spaces. This results in thrombosis, causing a retroplacental hematoma and leading to placental separation. Gestational diabetes, hyperemesis gravidarum, and a previous cesarean section are not specific risk factors for placental abruption.

The nurse is collecting data from a client who is pregnant with triplets. The client also has a 3-year-old child who was born at 39 weeks' gestation. The nurse would document which gravida and para status on this client?

Gravida I, para I 2Gravida II, para I 3Gravida II, para II 4Gravida III, para II 2 Gravida is a term that refers to a woman who is or who has been pregnant, regardless of the duration of the pregnancy. Parity is a term that means the number of births after 20 weeks' gestation; it does not reflect the number of fetuses or infants. Options 1, 3, and 4 are incorrect on the basis of these definitions.

The nurse is collecting data on a client who is pregnant with twins. Which signs would alert the nurse to potential problems specifically related to the twin pregnancy? Select all that apply.

Hypertension2Elevated blood glucose levels3Uterine size is large for gestational age4Six or more uterine contractions per hour 1,4 The mother with a multiple-gestation pregnancy is at a higher risk for preeclampsia and preterm labor than if she had a singleton pregnancy. The mother should be monitored for signs and symptoms of preeclampsia and preterm labor. A classic sign of preeclampsia is hypertension. Having 6 or more contractions per hour may indicate preterm labor. An elevated blood glucose level and Rh sensitization are concerns but are not unique to a multiple pregnancy. Uterine size may be large for gestational age in a multiple-gestation pregnancy.

The nurse is instructing a pregnant client in her first trimester about nutrition. The nurse would correct which misunderstanding on the part of the client about nutrition during pregnancy?

Iron supplements should be taken throughout pregnancy.2Calcium intake should be increased for the duration of the pregnancy.3Pregnancy greatly increases the risk of malnourishment for the mother.4The maternal diet significantly influences fetal growth and development. 3 Although pregnancy poses some nutritional risk for the mother, the client is not at risk for becoming malnourished. Intake of dietary iron is usually insufficient for the majority of pregnant women, and iron supplements routinely are encouraged. Calcium is critical during the third trimester but must be increased from the onset of pregnancy. Good nutrition during pregnancy significantly and positively influences fetal growth and development.

The nurse is gathering data from a pregnant client about physiological risk factors. The nurse would be sure to obtain which priority data?

Life stress2Self-care needs3Support systems4Weight and height 4 Height and weight are important factors to assess when determining physiological risk factors. Although life stress, self-care needs, and support systems are important to determine, they are not directly related to physiological risk factors.

The nurse is working with a woman who has just been diagnosed with gestational diabetes mellitus. The nurse informs the client of which issues that may occur during this pregnancy because of this condition? Select all that apply.

Low-birth-weight baby2Urinary tract infections3Increased chance of cesarean birth4Delayed lung maturation in the neonate5Pregnancy that lasts longer than 42 weeks 2,3,4 Women with gestational diabetes mellitus are at risk for gestational hypertension, preterm labor, hydramnios, both vaginal and urinary tract infections, an increased chance of cesarean birth or birth-related injuries, macrosomia in the neonate, and delayed lung maturation in the neonate

The nurse in a prenatal clinic is teaching a group of pregnant clients about anemia and foods high in iron. Which foods are high in iron content? Select all that apply.

Peanut butter2Fresh fruit salad3Whole grain bread4Omelet with cheese5Chocolate milkshake 1,3,4 Nuts such as peanuts are high in iron content. Whole grains and eggs are also high in iron content. Although dried fruits may be high in iron, fresh fruits are not. Milk has little, if any, iron content.

The nurse is reviewing the laboratory results of a pregnant client and notes that the hemoglobin level is decreased. Physiological dilutional anemia is documented in the client's record by the primary health care provider. The nurse plans care, knowing that this type of anemia is a result of which situation?

Poor intake of iron-rich foods2Decreased maternal hemoglobin formation3Decreased metabolism of iron during pregnancy4Increased blood volume of the mother during pregnancy 4 During the latter part of the first trimester, the blood volume of the mother increases more rapidly than blood cell production, leading to a decrease in the concentration of hemoglobin and erythrocytes. This is a normal process that causes a physiological anemia of pregnancy, or hemodilution. There is an increased metabolism of iron and maternal hemoglobin formation. The poor intake of iron-rich foods may cause an anemic condition, but physiological anemia of pregnancy occurs as a result of increased blood volume of the mother during pregnancy.

The nurse in a prenatal clinic is teaching a group of pregnant clients about physiological adaptations during pregnancy. Which are increased during the first trimester of pregnancy? Select all that apply.

Pulse2Blood volume3Cardiac output4Blood pressure5Red blood cell mass6White blood cell count 1,2,3,5In the first trimester of pregnancy the pulse increases 10 to 15 beats per minute, the blood volume increases 40% to 50%, the cardiac output increases 30% to 50%, and red blood cell mass increases 17%. Blood pressure decreases in the first half of pregnancy, returning to baseline in the second half. The white blood cell count increases in the second and third trimesters.

The nurse is reinforcing the positive effects of breathing and relaxation techniques to a pregnant client with cardiac issues who has an 18-month-old child. Which primary outcome is the purpose for these interventions?

Reducing maternal stress and fatigue2Helping the client prepare for labor and delivery3Avoiding stress-induced infectious disease processes4Preparing for maternal-child separation during hospitalization 3 A variety of factors can cause increased emotional stress during pregnancy, resulting in further cardiac complications. The client with known cardiac disease is at greater risk for such complications. The strategies identified in this question would primarily reduce maternal stress and fatigue. Although the other options identify possible outcomes, they are not the primary purpose of these strategies.

During an initial prenatal visit, the nurse notes that the primary health care provider documents that the client is experiencing iron deficiency anemia. Which client data support this finding? Select all that apply.

Reports of fatigue2Pink mucous membranes3Increased vaginal secretions4Hemoglobin level of 10.2 g/dL5Increased frequency of voiding 1,2 Anemia is a common problem in pregnancy and is characterized by a hemoglobin level of less than between 10.5 and 11 g/dL. Iron deficiency anemia and folic acid deficiency are two common types of anemia that present a concern during pregnancy. Although fatigue may be seen in some pregnant women, its presence may reflect complications caused by decreased oxygen supply to vital organs, thus supporting the laboratory findings. The other options are normal observations during pregnancy.

A pregnant client who has a positive pulmonary identification of the tuberculosis (TB) organism has been prescribed both isoniazid and rifampin. The nurse plans to implement which intervention?

Reviewing daily nutritional intake with the client2Reinforcing that infants are usually not susceptible to TB infection3Informing the client that follow-up care after delivery will not be needed4Encouraging the client to stop taking medications during the last trimester of pregnancy 1 Social conditions placing pregnant women at risk for TB include poverty, crowded living conditions, and malnutrition. In the case of acute disease during the antenatal period, a 9-month course of isoniazid and rifampin is suggested. Follow-up sputum screenings and evaluations are essential to establish treatment effectiveness after delivery. Teaching the client about the importance of an adequate nutritional intake needs to be included in the home care instructions. The remaining options do not contain correct information. Infants are susceptible to TB, follow-up care following delivery is needed, and the client needs to take the medications as prescribed

The nurse working in a prenatal clinic reviews a client's chart and notes that the primary health care provider documents that the client has a gynecoid pelvis. The nurse plans care understanding that which findings are characteristic of this type of pelvis? Select all that apply.

Round shape2Shallow depth3Narrow pubic arch4Diagonal conjugate measures 12.5 cm to 13 cm5Blunt, somewhat widely separated ischial spines 1,4,5 A gynecoid pelvis is a normal female pelvis, and it is the most favorable for successful labor and birth. Characteristics of a gynecoid pelvis include a round shape, blunted ischial spines that are widely separated, a diagonal conjugate of at least 12.5 cm to 13 cm, a wide pelvic arch, and an adequate depth.

The nurse is reading the primary health care provider's (PHCP) documentation regarding a pregnant client and notes that the PHCP has documented that the client has a platypelloid pelvic shape. The nurse recognizes which characteristics to be present in the platypelloid pelvis? Select all that apply.

Shallow depth2Wide suprapubic arch3Deep, curved sacral area4Compatible with vaginal delivery5Flattened anteroposteriorly and wide transversely 1,2,4,5 A gynecoid pelvic shape is rounded with a wide pubic arch and is the most favorable pelvic shape for a vaginal birth. The platypelloid pelvic shape has a shallow depth, wide suprapubic arch, may be compatible with vaginal delivery, and has a flattened anteroposterior diameter. The gynecoid pelvis is the only pelvic type to have a deep, curved sacral area

The nurse reviews the client's health record and notes that based on Leopold's maneuvers, the fetus is in a cephalic presentation. Which findings while performing Leopold's maneuvers support the identification of a cephalic presentation? Select all that apply.

Small parts are located on the left side of the uterus.2Small parts are located on the right side of the uterus.3A round hard ballottable shape is located in the fundus.4A round hard ballottable shape is located just above the symphysis pubis.5A soft, irregular non-ballottable shape is located just above the symphysis pubis. 1,2,5 The cephalic presentation is more favorable than others and is the most common. Abnormal presentations result in prolonged labor and are likely to necessitate a cesarean birth. Small parts may be located on either side of the uterus. In the cephalic presentation, the round, ballottable head is located just above the symphysis pubis. If the round, ballottable shape is located in the fundus or if a soft, irregular shape is present over the symphysis pubis, the fetus is in a breech presentation.

A pregnant client is a gravida III, para 0, abortus II. She is placed on bed rest at home because of preterm labor. The nurse provides information to the husband, knowing that which instruction will assist in promoting family adaptation?

Teaching the husband to administer and titrate tocolytic agents2Teaching the husband to instruct the wife how to perform Kegel exercises3Telling the husband that sexual intercourse has probably caused the preterm labor4Teaching the husband to perform passive range of motion and provide back rubs for his wife 4 Range-of-motion exercises will help maintain muscle tone during bed rest, and back rubs provide skin-to-skin contact and are comforting. The inclusion of the significant other promotes adaptation and decreases the sense of isolation. Telling the husband that sexual intercourse has led to the preterm labor will lead to guilt and maladaptation. The husband should not be expected to titratemedications. Kegel exercises are beneficial but will not provide the human-to-human contact that promotes family adaptation.

The nurse is providing emotional support to a client who experienced a spontaneous abortion. The nurse can best assist the client by planning care that focuses on which psychosocial issue?

The feelings of guilt that is often associated with grief2Grief and loss are usually resolved within 3 months3The amount of pain and discomfort as a result of the abortion4The other children in the family and the ability to bear children in the future 1 Nurses must consider the psychological needs of the family experiencing spontaneous abortion. Grief often includes feelings of guilt. The grieving process is individual and may last a year or longer. The amount of pain and discomfort is important, but this is a physiological concern. It is not appropriate to focus on the client's ability to have other children.

The nurse is providing education to a client with gestational diabetes who was recently placed on insulin therapy. Which information would the nurse tell the client about insulin needs during the second and third trimesters of pregnancy?

The insulin needs will increase.2The insulin needs will decrease.3The insulin needs will remain unchanged.4The client will require both short- and long-term insulin therapy. 1 Insulin needs in the second and third trimesters of pregnancy increase. Insulin dosages must be adjusted to balance changes in caloric intake and other factors that can increase insulin needs (e.g., illness, trauma, stress, adolescent growth spurt, pregnancy after the first trimester). Stating that the insulin needs will decrease or remain unchanged, or that the client will require both short- and long-term insulin therapy, is incorrect.

The nurse is collecting data from a pregnant client when the client asks the nurse about the purpose of the fallopian tubes. Which is the accurate response the nurse would make?

The organ of copulation2Where the fetus develops3Where fertilization occurs4The organ that secretes estrogen and progesterone 3 Each fallopian tube is a hollow muscular tube that transports a mature oocyte for final maturation and fertilization. Fertilization typically occurs near the boundary between the ampulla and the isthmus of the tube. The vagina is the organ of copulation, and the fetus develops in the uterus. Estrogen is a hormone that is produced by the ovarian follicles, the corpus luteum, the adrenal cortex, and the placenta during pregnancy. Progesterone is a hormone that is secreted by the corpus luteum of the ovary, the adrenal glands, and the placenta during pregnancy.

The nurse is discussing prenatal testing with a woman who is approximately 6 weeks pregnant. The nurse shares which tests are expected to be conducted during the first trimester? Select all that apply.

Urinalysis2Rubella titer3Blood glucose4Complete blood count5Serum alpha-fetoprotein 1,2,4 The expected routine prenatal testing during the first semester includes blood type and Rh factor and antibody screen, a complete blood count, a VDRL (rapid plasma reagin), a rubella titer, human immunodeficiency virus (HIV) screen, urinalysis and culture, Pap test, and a cervical culture. Serum alpha-fetoprotein is a routine test conducted in the second trimester. Blood glucose screening may not occur until the third trimester.

A pregnant client in the third trimester of pregnancy with a diagnosis of mild preeclampsia is being monitored at home for progression of the disease process. The home care nurse reinforces teaching the client about the signs that need to be reported to the primary health care provider (PHCP) and tells the client to call the PHCP if which occurs?

Urine output increases.2Fetal movements are more than 4 per hour.3Weight increases by more than 1 pound in a week.4The blood pressure reading is between 122/80 and 138/88 mm Hg. 3 The nurse should instruct the client to report any increase in blood pressure, protein in the urine, weight gain greater than 1 pound per week, or edema. The client also is taught how to count fetal movements and is instructed that decreased fetal activity (three or fewer movements per hour) may indicate fetal compromise and should be reported.

A client is a gravida IV, para III in her final trimester of pregnancy. She does not attend usual social functions because of the fear of stress incontinence. Her oldest child is in a school play, which she wants to attend. Which measure is appropriate to suggest to the client?

Wear a perineal pad to the play.2Have a friend videotape the play for her.3Perform Kegel exercises during the play.4Limit fluid intake to 500 mL on the day of the play. 1 Wearing a perineal pad will give the client the security that she needs. The client should be instructed to remove a damp pad as soon as possible to decrease the incidence of infection. A videotape will not satisfy the client's need to be present at the play. Kegel exercises are useful to promote long-term bladder tone but will not be effective with one day's use. Limiting fluid intake can be harmful.

A client who has just been told that she is pregnant asks a clinic nurse when the fetus's heart will be developed and beating. The nurse tells the client that the fetal heart is beating at what gestational week?

Week 5 2. Week 6 3. Week 7 4. Week 8 1 The fetal heart is beating and has developed four chambers by gestational week 5.

The nurse reviews the antenatal history of a client in early labor. The nurse recognizes that which factor noted in the history presents the greatest potential for causing neonatal sepsis following delivery?

Weight gain of 25 to 35 pounds2Prenatal care beginning at 8 weeks3Spontaneous rupture of membranes 2 hours ago4History of substance abuse during this pregnancy 4 Risk factors for neonatal sepsis can arise from maternal, intrapartal, or neonatal conditions. Maternal risk factors before delivery include low socioeconomic status, poor prenatal care and nutrition, and a history of substance abuse during pregnancy. Weight gain of 25 to 35 pounds is acceptable for a woman of average nonpregnant weight. Prenatal care beginning in the first trimester is not an added risk factor. Premature rupture of the membranes or prolonged rupture of membranes greater than 18 hours before birth is also a risk factor for neonatal acquisition of infection.

The cephalic presentation is more favorable than others and is the most common. Abnormal presentations result in prolonged labor and are likely to necessitate a cesarean birth. Small parts may be located on either side of the uterus. In the cephalic presentation, the round, ballottable head is located just above the symphysis pubis. If the round, ballottable shape is located in the fundus or if a soft, irregular shape is present over the symphysis pubis, the fetus is in a breech presentation.

"A newborn cannot contract HIV during delivery."2"There is no risk to the newborn of an HIV-infected mother during delivery."3"Newborns who contract HIV during delivery will show immediate symptoms."4"There is a risk of transmission from HIV-positive mothers to their newborn, although the newborn may be asymptomatic at birth." 4 There is a risk of transmission of HIV to a newborn at the time of delivery if the pregnant woman is HIV positive. With appropriate pharmacologic intervention for the HIV-positive mother, transmission of the virus to the newborn has minimal risk. Newborns may not exhibit symptoms for 18 months or more.

The nursing instructor asks the nursing student to identify the reason that the pulse rate of a client in the second trimester of pregnancy has increased since the last visit. Which response indicates that the student understands the rationale of this physiological response?

"An increase in pulse relates to the development of preeclampsia."2"Blood volume and cardiac output increase resulting in a faster pulse."3"The pulse rate is an emotional response to the excitement she is experiencing at being pregnant."4"There should not be an increase in pulse; therefore, the change is most likely related to cardiac malfunction." 2 Between 14 and 20 weeks of gestation, the pulse increases slowly, up 10 to 15 beats per minute, which lasts until term. Cardiac output and blood volume increase. Blood pressure decreases not increases during the first half of pregnancy, returning to baseline in the second half of pregnancy. Although excitement may cause an increase in pulse rate, the likely cause is the combination of normal physiological changes that occur during pregnancy. An increase in the pulse rate is not due to a cardiac malfunction. The remaining options are not supported by the information given in the question.

The nurse is caring for a pregnant client who was diagnosed with acquired immunodeficiency syndrome (AIDS) and asks the nurse if she will be able to breastfeed the infant after delivery. Which response by the nurse is appropriate?

"Breastfeeding is contraindicated."2"Breastfeeding is allowed as long as the mother is taking zidovudine (AZT)."3"Breastfeeding is allowed as long as the infant receives an immunization for HIV."4"Breastfeeding is allowed as long as the infant is not showing signs of human immunodeficiency virus (HIV) infection." Correct Answer1"Breastfeeding is contraindicated."2"Breastfeeding is allowed as long as the mother is taking zidovudine (AZT)."3"Breastfeeding is allowed as long as the infant receives an immunization for HIV."4"Breastfeeding is allowed as long as the infant is not showing signs of human immunodeficiency virus (HIV) infection." SubmitCorrect 1 The woman diagnosed with AIDS will need to know that breastfeeding is contraindicated but that she can provide all other care for her infant. Characteristically, the newborn is asymptomatic at birth, and signs and symptoms usually become obvious during the first year of life. No immunization is available for HIV.

A young pregnant woman with diabetes mellitus has lost 10 pounds during the first 15 weeks of gestation. The client tells the nurse, "I do not eat regular meals." Based on the client's statement, which is the best response by the nurse?

"Can you tell me more about what you are eating?"2"If you do not eat regular meals, you will hurt your baby."3"It does not matter anymore how much weight you gain."4"I'll have the primary health care provider review your diet history." 1 It is important for the nurse to obtain additional information from the client. In option 1, the nurse is using the therapeutic communication tool of validation and clarification in order to obtain more information. The other options will block communication. Option 2 devalues the client and shows disapproval. Option 4 is avoiding the subject, and option 3 provides false reassurance.

A woman at 20 weeks of gestation calls the primary health care provider's office and speaks to the nurse. The client states that she is having subtle but persistent changes in her vaginal discharge, menstrual-like cramps, and diarrhea. Which is the least helpful response to the client?

"Drink three glasses of water and lie on your left side for 1 hour."2"This is an emergency; you should come to the clinic within the hour."3"Tell me about your activity, food, fluid, and medication intake for the past 24 hours."4"Palpate for contractions and if four or more are felt within 1 hour, you need to be seen by the primary health care provider." 2 The nurse needs to determine if this client is experiencing preterm labor or uterine irritability. The woman should be instructed to lie on her side, drink fluids, and keep her bladder empty. This will decrease uterine activity and prevent uterine hypoxia. Option 3 addresses the process of data collection and is an important initial component of care. If the woman continues to have persistent uterine activity after 1 hour or counts four or more contractions in less than an hour, she should be seen for further evaluation. Telling the client that this is an emergency without first attempting the other options places undue stress on the pregnant client.

The nurse-midwife is conducting a session on the process of conception with a group of nursing students. Which statements reflect that the nursing students understand the process of conception? Select all that apply.

"Fertilization occurs in the outer third of the fallopian tube."2"Only 1 sperm will penetrate the ovum to produce fertilization."3"The pre-embryonic period is defined as the first 8 weeks of gestation."4"Implantation occurs in the anterior or posterior fundal region of the uterus."5"The ovary produces hormones to maintain the pregnancy before placental development." 1,2,4,5 Fallopian tubes, also called oviducts, are 8 to 14 cm long and are quite narrow. The fallopian tubes are a pathway for the ovum between the ovary and the uterus. Fertilization occurs in the outer third of the fallopian tube. After the sperm penetrates the ovum, a membrane is formed that blocks the entrance of additional sperm. Implantation usually occurs in the anterior or posterior fundal region. The corpus luteum found in the ovary is responsible for producing early hormones which maintain the pregnancy. The pre-embryonic period lasts until day 15.

A couple comes to the family planning clinic and asks about sterilization procedures. Which question by the nurse helps determine whether this method of family planning is appropriate?

"Have either of you ever had surgery?"2"Do you plan to have any other children?"3"Do either of you have diabetes mellitus?"4"Do either of you have problems with high blood pressure?" 2 Sterilization is a method of contraception for couples who have completed their families. It should be considered a permanent end to fertility because reversal surgery is not always successful. The nurse would ask the couple about their plans for having children in the future. Options 1, 3, and 4 are unrelated to this procedure.

When collecting data on a pregnant client, the nurse includes which question to determine whether the client is at risk for toxoplasmosis parasite infection?

"Have you been sexually active during the pregnancy, and if so, with how many different partners?"2"Have you experienced any high fevers or unusual rashes during the first 6 weeks of your pregnancy?"3"Do you have any cats as house pets, and if so, do you ever come in contact with their soiled kitty litter?"4"Have you been recently exposed to children with draining skin rashes or gastrointestinal symptoms?" 3 Toxoplasmosis is a systemic, usually asymptomatic illness caused by a protozoal parasite. Humans acquire the infection from inadequately cooked meat, eggs, or milk or from ingesting or inhaling the oocyst stage of the parasite excreted in feline feces in contaminated soil or kitty litter. The remaining options are not related to this disease.

A pregnant client tells the nurse that she has been experiencing pain as a result of hemorrhoids. Which statement by the client identifies the need for further teaching regarding the hemorrhoids?

"Hemorrhoids can be gently pushed back inside my body using a lubricant."2"Diet is very important in the treatment of hemorrhoids. Plenty of liquids and a balance of bulk in the diet are needed."3"Hemorrhoids are aggravated by standing for long periods. I need to lie down periodically during the day to relieve the pressure."4"Hemorrhoids are caused solely by the changes in hormones during pregnancy. They will go away within a day or two after the baby is born." 4 Hemorrhoids are varicosities and are most likely precipitated during pregnancy by the pressure of the growing fetus inside the abdominal cavity, iron consumption, and constipation. Standing aggravates the problem. Dietary factors such as fluids and roughage and the technique of manual reduction should be included in the plan of care. Hormonal changes are not the only factor to be considered.

A pregnant client tests positive for hepatitis B virus (HBV). The nurse determines that the client understands this infection when the client makes which statement?

"Hepatitis B will cause a severe eye infection in my baby."2"I know my baby will be immune from hepatitis for the first 2 months of life."3"I am so glad that I can breastfeed my baby after she has been vaccinated."4"I feel sad that my baby is going to be isolated in the nursery after my delivery." 3 To reduce the possibility of hepatitis B virus being spread to the newborn, neonates routinely are vaccinated at birth. Although HBV is transmitted in breast milk, once serum immune globulin has been administered, the mother may breastfeed without risk to the newborn. Option 2 is incorrect because the baby is not provided immunity to this condition. HBV does not cause a severe eye infection in the newborn, and this newborn does not need to be isolated.

During a routine prenatal visit, a client complains of gingivitis and gums that bleed easily with brushing. When assisting to plan the care for the client, the nurse includes a goal that addresses proper nutrition to minimize this problem. The nurse determines that the goal has been achieved when the client makes which statement?

"I am drinking 8 ounces of water with each meal."2"I eat two saltine crackers before I get up each morning."3"I am eating three servings of cracked-wheat bread each day."4"I am eating fresh fruits and vegetables for snacks and for dessert each day." 4 Fresh fruits and vegetables will provide vitamins and minerals needed for healthy gums. Drinking water with meals has no direct effect on gums; saltine crackers before arising helps decrease nausea. Cracked-wheat bread may abrade the tender gums.

The nurse in the prenatal clinic is taking a nutritional history from a pregnant adolescent. Which statement by the client would alert the nurse to a potential concern regarding adequate nutritional intake during the pregnancy?

"I am not crazy about eating vegetables, but I will do my best."2"I need to gain only 10 pounds so that my baby will be small like I am."3"I don't like milk, but I can drink it if it is in a shake mixed with chocolate."4"I really like to have a root beer float with vanilla ice cream in the afternoon." 2 Pregnant adolescents are at higher risk for complications than are other pregnant clients. Adolescents are often concerned about their body image. If weight is a major focus, the adolescent is more likely to restrict calories to avoid weight gain. Only gaining 10 pounds, which is much too restrictive regarding weight gain, is the only response that suggests a possible concern. Option 1 expresses an attempt to consume required vegetables. Options 3 and 4 indicate that the client will consume items that will help increase calcium intake.

A pregnant client who has gestational diabetes mellitus tells the nurse that she is concerned about what her baby's condition will be following delivery. Which nursing response best supports the client?

"I am sure your baby will be fine."2"You will not have any problems if you keep your blood sugar in control."3"Your baby will need to spend most of the time in the nursery after delivery."4"Better blood glucose control means fewer effects; let's review your plan of care." 4 When the maternal blood glucose is high, the fetus produces more insulin, which is a process that continues after delivery. Keeping the maternal glucose in better control lessens this effect. Options 1 and 2 provide a false reassurance to the client. Option 3 will cause further concern in the client. Option 4 provides the most realistic support for the client and allows the nurse an opportunity to review the client's plan of care to clarify information and reassure the mother.

During a prenatal visit, the nurse is explaining dietary management to a client with diabetes mellitus. The nurse determines that the teaching has been effective when the client makes which statement?

"I can eat more sweets now because I need more calories."2"I need more fat in my diet so that the baby can gain enough weight."3"I need to eat a high-protein, low-carbohydrate diet now to control my blood glucose."4"I need to increase the fiber in my diet to control my blood glucose and prevent constipation." 4 An increase in calories is needed during pregnancy, but concentrated sugars should be avoided because they may cause hyperglycemia. Per health care provider recommendations, fat intake should be 20% to 30% of the total calories. In addition, the client with diabetes needs about 50% to 60% of her caloric intake from carbohydrates and about 12% to 20% from protein. High-fiber foods will control blood glucose levels and prevent constipation.

The nurse is gathering data from a 16-year-old pregnant client during her initial prenatal clinic visit. The client is beginning week 18 of her first pregnancy. Which client statement indicates a need for further investigation?"

"I don't like my figure anymore. My clothes are all too tight."2"I don't like my breasts anymore. These silver lines are ugly."3"I don't like my stomach anymore. That brown line is disgusting."4"I don't like my face anymore. I always look like I have been crying." 4 Options 1, 2, and 3 are dealing with body image. Although these comments should not be ignored, the need for follow-up is not urgent. Option 4 is an implication of periorbital and facial edema, which could be indicative of gestational hypertension (GH). Because this is an adolescent who has not sought early prenatal care, she is at higher risk for the development of gestational hypertension.

The nurse is reinforcing instructions to a pregnant client regarding dietary measures to promote a healthy pregnancy. The nurse instructs the client to consume an adequate intake of fluid on a daily basis. Which statement by the client indicates an understanding of the daily fluid requirement?

"I should drink 8 to 12 glasses of liquid in addition to my daily milk requirement."2"I should drink 8 to 12 glasses of liquid a day, and I can count the coffee that I drink."3"I should drink 8 to 12 glasses of liquid a day, and I can count the tea, fruit juices, or milk that I drink."4"I should drink 8 to 12 glasses of liquid a day, and I can count the carbonated soft drinks that I consume." 1 The nurse should instruct the client to drink an adequate fluid intake on a daily basis to assist in digestion and in the management of constipation: 8 to 12 glasses of liquids (1500 to 2000 mL) in addition to the daily milk requirement are recommended every day. This fluid should be water or fruit and vegetable juices rather than carbonated soft drinks or caffeinated beverages.

The nurse collects data from a pregnant client diagnosed with iron deficiency anemia during her third trimester for additional risk factors associated with the anemia. Which statement made by the client would the nurse question to receive more information?

"I will drink at least 6 to 8 glasses of water each day."2"I will take a nap each afternoon to help me feel more rested."3"I have had mild vaginal spotting twice since my last prenatal visit."4"I will continue to take the extra iron that was prescribed for me by the primary health care provider." 3 "I will drink at least 6 to 8 glasses of water each day."2"I will take a nap each afternoon to help me feel more rested."3"I have had mild vaginal spotting twice since my last prenatal visit."4"I will continue to take the extra iron that was prescribed for me by the primary health care provider."

The nurse has a teaching session with a malnourished client regarding iron supplementation to prevent anemia during pregnancy. Which statement indicates successful learning?

"Iron supplements will give me diarrhea."2"The iron is needed for the red blood cells."3"Meat does not provide iron and should be avoided."4"My body has all the iron it needs and I don't need to take supplements." 2 A nutritional supplement that is commonly needed during pregnancy for the red blood cells is iron. Anemia in pregnancy is primarily caused by iron deficiency. Iron supplements usually cause constipation. Meats are an excellent source of iron. Iron for the fetus comes from the maternal serum.

The nursing instructor asks a nursing student to describe the process of quickening. Which statement indicates an understanding of this term?

"It is the fetal movement that is felt by the mother."2"It is the compressibility of the lower uterine segment."3"It is the irregular, painless contractions that occur throughout pregnancy."4"It is the soft blowing sound that can be heard when the uterus is auscultated." 1 Quickening is fetal movement that appears usually at weeks 16 to 20, when the expectant mother first notices subtle fetal movements that gradually increase in intensity. A compressibility of the lower uterine segment occurs at about 6 weeks' gestation and is called Hegar's sign. Braxton Hicks contractions are irregular, painless contractions that may occur throughout pregnancy. A soft blowing sound that corresponds with the maternal pulse may be auscultated over the uterus; this is known as uterine souffle. This sound is the result of blood circulation to the placenta, and it corresponds with the maternal pulse.

The nurse is teaching a pregnant woman about the physiological effects and hormone changes that occur in pregnancy. The woman asks the nurse about the purposes of estrogen. Which responses would the nurse make to the client? Select all that apply.

"It maintains the uterine lining for implantation."2"It prevents the involution of the corpus luteum."3"It stimulates the breasts to prepare for lactation."4"It stimulates metabolism of glucose and converts the glucose to fat."5"It maintains the production of progesterone until the placenta is formed."6"It stimulates uterine development to provide an environment for the fetus." 3,6 Estrogen stimulates uterine development to provide an environment for the fetus and stimulates the breasts to prepare for lactation. Progesterone maintains the uterine lining for implantation and relaxes all smooth muscle. Human chorionic gonadotropin prevents involution of the corpus luteum and maintains the production of progesterone until the placenta is formed. Human placental lactogen stimulates the metabolism of glucose and converts the glucose to fat; it is antagonistic to insulin.

The nurse working in a prenatal clinic receives a telephone call from a client at 22 weeks of gestation. The client reports some vaginal discharge and has started to experience menstrual-like cramps and diarrhea. Which responses by the nurse indicate an understanding of the implications of the client's signs/symptoms? Select all that apply.

"Lie on your left side for an hour and try to drink some fluids."2"It is important that you urinate frequently to keep your bladder empty."3"This is probably an emergency. Have someone drive you to a hospital now."4"Palpate for contractions and call back if there are more than four contractions in the next hour."5"Can you identify what you ate and drank, what medications you took, and your activity during the past 24 hours?" 1,2,4,5 If a client experiences uterine activity, it may be helpful to have her lie on the left side and drink fluids to reduce uterine hypoxia and activity. It may also be helpful to keep the bladder empty. If the woman continues to have persistent uterine activity after 1 hour or counts four or more contractions in less than an hour, she should be seen for further evaluation. The information presented in the question does not represent an emergency at this time, but it requires further monitoring.

Which statements made by a nursing student indicate that the student has an appropriate knowledge base regarding the pregnancy hormone human chorionic gonadotropin (hCG)? Select all that apply.

"Maximum level of human chorionic gonadotropin is reached at term."2"Human chorionic gonadotropin is the hormone responsible for a positive pregnancy test."3"Human chorionic gonadotropin may be present as early as 8 to 10 days following conception."4"Human chorionic gonadotropin is produced by the trophoblastic cells that surround the developing embryo."5"Human chorionic gonadotropin preserves the function of the ovarian corpus luteum so that estrogen and progesterone are produced before placental functioning." 2,3,4,5 Human chorionic gonadotropin may be responsible for some of the nausea and vomiting associated with early pregnancy. Human chorionic gonadotropin is the hormone responsible for positive pregnancy tests. In early pregnancy as early as 8 to 10 days following conception, hCG is produced by trophoblastic cells that surround the developing embryo. This hormone is also responsible for maintaining the ovarian corpus luteum, which supplies estrogen and progesterone to help maintain the pregnancy until the placenta is fully functioning. Maximum levels of hCG are present from 50 to 70 days into the pregnancy and then this level declines.

The nurse is assisting in conducting a prenatal session with a group of expectant parents. Which comment related to female hormones made by a parent indicates the need for further teaching?

"Oxytocin may be used to stimulate labor contractions."2"Prolactin is the hormone responsible for the initiation of labor."3"Progesterone plays a role in preparing the uterus for embryo implantation."4"Testosterone in the female helps stimulate pubic and axillary hair growth at puberty." 2 Prolactin stimulates the secretion of milk, called lactogenesis. Oxytocin stimulates contractions during birth and stimulates postpartum contractions to compress uterine vessels and control bleeding. Progesterone stimulates the secretions of the endometrial glands, causing endometrial vessels to become highly dilated and tortuous in preparation for possible embryo implantation.

A nursing instructor instructs the nursing students that surfactant is a substance needed to facilitate neonatal breathing. Which statements made by the nursing students indicate understanding regarding the presence of surfactant? Select all that apply.

"Surfactant is manufactured by the fetal adrenal glands."2"Surfactant is necessary to enhance clotting in the newborn."3"Surfactant, which is needed for lung expansion, is present beginning at 28 weeks."4"With decreased surfactant, more pressure must be generated to produce inspiration."5"Surfactant lowers surface tension, reducing the pressure required to keep the alveoli expanded." 3,4,5 The alveoli of the term infant's lungs are lined with surfactant. Surfactant, a substance needed to facilitate neonatal breathing, begins to be produced at approximately week 28. When surfactant is decreased or absent, more pressure will be needed to produce and maintain inspiration. Surfactant is responsible for lowering surface tension, which allows the alveoli to more easily remain open. Surfactant is produced by type 2 lung cells and is not a part of the clotting mechanism for the newborn.

A client is scheduled for an amniocentesis and tells the nurse, "I'm not sure I should have this test done." Which response by the nurse is appropriate?

"Tell me what concerns you have."2"Don't worry. Everything will be fine."3"Why don't you want to have this test done?"4"The primary health care provider has scheduled this test for a reason." 1 The nurse needs to gather more data and assist the client in exploring her feelings about the test. Options 2, 3, and 4 are blocks to communication and are nontherapeutic nursing responses.

A pregnant woman visiting a health care clinic for the first prenatal visit hears the primary health care provider discuss the preembryonic period of development with the nurse. The woman asks the nurse what this means. What information would the nurse share related to this stage of development? Select all that apply.

"The preembryonic period is the period of time before conception."2"The preembryonic period is the longest period of fetal development."3"The preembryonic period is the first 2 weeks of fetal development following conception."4"The preembryonic stage is the most critical time in the development of the organ systems and the main external features."5"The preembryonic period is the fetal development period from the beginning of the third week through the eighth week after conception."6"The preembryonic period includes initial development of the embryonic membranes and establishment of the primary germ layers." 3,6 he preembryonic period is the first 2 weeks after conception. Around the fourth day after conception, the fertilized ovum, now called a zygote, enters the uterus. The preembryonic period includes blastocyst formation, initial development of the embryonic membranes, and establishment of the primary germ layers. The embryonic period of development extends from the beginning of the third week through the eighth week after conception. Basic structures of all major body organs are completed during the embryonic period. The fetal period is the longest part of prenatal development. It begins 9 weeks after conception and ends with birth. All major systems are present in their basic form.

A nonstress test is prescribed for a pregnant client, and the client asks the nurse about the procedure. Which informative statement would the nurse provide to the client?

"The test is an invasive procedure and requires that you sign an informed consent."2"The fetus is challenged by uterine contractions to obtain the necessary information."3"The test will take about 2 hours and will require close monitoring for 2 hours after the procedure is completed."4"An ultrasound transducer that records fetal heart activity is secured over the abdomen where the fetal heart is heard most clearly." 4 A nonstress test is a noninvasive test, and an ultrasound transducer that records fetal heart activity is secured over the maternal abdomen where the fetal heart is heard most clearly. A tocodynamometer that detects uterine activity and fetal movement is then secured to the maternal abdomen. Fetal heart activity and movements are recorded. The test is termed "nonstress" because it consists of monitoring only; the fetus is not challenged or stressed by uterine contractions to obtain the necessary data. The nonstress test takes about 30 to 40 minutes.

A contraction stress test is scheduled for the client. The woman asks the nurse about the test. Which response describes the most accurate description of the test?"

"Uterine contractions are stimulated by Leopold's maneuvers."2"The uterus is stimulated to contract by either small amounts of oxytocin or by nipple stimulation."3"An internal fetal monitor is attached, and you will walk on a treadmill until contractions begin."4"Small amounts of oxytocin are administered during internal fetal monitoring to stimulate uterine contractions." 2 A contraction stress test assesses placental oxygenation and function and determines the fetus's ability to tolerate labor, as well as its well-being. The test is performed if the nonstress test result is abnormal. During the stress test, the fetus is exposed to the stressor of contractions to assess the adequacy of placental perfusion under simulated labor conditions. An external fetal monitor is applied to the mother, and a 20- to 30-minute baseline strip is recorded. The uterus is stimulated to contract, either by the administration of a dilute dose of oxytocin or by having the mother use nipple stimulation, until three palpable contractions with a duration of 40 seconds or more during a 10-minute period have occurred. Frequent maternal blood pressure readings are performed and the client is monitored closely while increasing doses of oxytocin are given. Leopold's maneuvers are performed to locate the position of the fetus. Only external fetal monitoring is employed.

The nurse shares with a pregnant client that the result of her rubella screening is positive. Which is the nurse's response when asked by the client if it is safe for her 15-month-old toddler to receive the rubella vaccine?

"You are still susceptible to rubella, so your toddler should receive the vaccine."2"Children do not receive the rubella vaccine until they have had their fifth birthday."3"It is discouraged that children of pregnant women be vaccinated during the pregnancy."4"You are immune to the virus so it is safe for your toddler to receive the vaccine at this time." 4 A positive maternal titer further indicates that a significant antibody titer has developed in response to a prior exposure to Rubivirus; therefore, immunity to the virus has been achieved, so it is safe to be exposed as through contact with a newly vaccinated child. Children should receive their rubella immunization according to schedule (12 to 15 months of age).

A pregnant woman in her second trimester calls the prenatal clinic nurse to report a recent exposure to a child with rubella. Which response by the nurse is appropriate?

"You should avoid all school-age children during pregnancy."2"There is no need to be concerned if you don't have a fever or rash within the next 2 days."3"Be sure to tell the doctor on your next prenatal visit, but there is little risk in the second trimester."4"You were wise to call. I will check your rubella titer screening results, and we can identify immediately if interventions are needed." 4 Rubella virus is spread by aerosol droplet transmission through the upper respiratory tract and has an incubation period of 14 to 21 days. Rubella titer screening is a standard antenatal test for women during their initial screening. The results of this screening test need to be checked to determine if interventions are necessary. Options 1, 2, and 3 are inappropriate statements and do not address the subject of the question.

A client with type 1 diabetes mellitus in the first trimester of pregnancy is scheduled for a primary health care provider's visit. The client asks the nurse whether a change in the medication to treat the diabetes will occur at this time. Which statement is true?

"You will only use insulin per sliding scale."2"Your normal insulin dosage will have to be decreased."3"Your NPH insulin dosage before supper will need to be increased."4"You will not have to increase your normal insulin dosage at this time." 2 Typically, insulin dosage may have to be reduced to avoid hypoglycemia in the first trimester when nausea decreases appetite and physical activity may be reduced. A prescribed amount of insulin may be administered with additional Insulin being given on a sliding scale in which the woman varies her dose of insulin based on blood glucose levels. NPH insulin is not given at dinnertime because hypoglycemia during the night can occur.

A client in the first trimester of pregnancy arrives at the health care clinic and reports that she has been experiencing vaginal bleeding. A threatened abortion is suspected, and the nurse reinforces a list of instructions for the client regarding management of care. Which instructions would the nurse place on the list? Select all that apply.

.To note the color of blood on each perineal pad2To watch for the evidence of the passage of tissue3To note the quantity of blood on each perineal pad4To count the number of perineal pads used on a daily basis5To avoid any sexual activity for the remainder of the pregnancy 1,2,3,4

The nurse is collecting data from a client who is pregnant with twins. The client has a healthy 5-year-old child who was delivered at 38 weeks, and she tells the nurse that she does not have a history of any type of abortion or fetal demise. The nurse would document which as the GTPAL for this client?

1. G = 3, T = 2, P = 0, A = 0, L = 1 2. G = 2, T = 1, P = 0, A = 0, L = 1 3. G = 1, T = 1, P = 1, A = 0, L = 1 4. G = 2, T = 0, P = 0, A = 0, L = 1 2 Pregnancy outcomes can be described with the GTPAL acronym: G = gravidity (number of pregnancies); T = term births (number born after 37 weeks); P = preterm births (number born before 37 weeks' gestation); A = abortions/miscarriages (number of abortions/miscarriages); L = live births (number of live births or living children). Therefore, a woman who is pregnant with twins and who already has a child has a gravida of 2. Because the child was delivered at 38 weeks, the number of preterm births is 0, and the number of term births is 1. The number of abortions is 0, and the number of live births is 1.

The nurse is reviewing the health care record of a pregnant client at 24 weeks' gestation. The nurse would anticipate that the fundus would be located at which level?

16 cm to 18 cm 2. 20 cm to 22 cm 3. 22 cm to 26 cm 4. 32 cm to 36 cm 3 At 12 weeks' gestation, the uterus extends out of the maternal pelvis and can be palpated above the symphysis pubis. At 24 weeks, the fundus should be located plus or minus 2 cm of the gestational age.

A pregnant client asks the nurse in the clinic when she will be able to start feeling the fetus move. The nurse responds by telling the mother that fetal movements will be noted between which weeks of gestation?

6 and 8 weeks' gestation 2. 8 and 10 weeks' gestation 3. 10 and 12 weeks' gestation 4. 16 and 20 weeks' gestation 4 Quickening is fetal movement that usually first occurs between 16 and 20 weeks' gestation. The expectant mother first notices subtle fetal movements during this time, and these gradually increase in intensity. Options 1, 2, and 3 are incorrect; these gestational time frames are too early for quickening.

A blood glucose screening measurement is performed on a pregnant client, and the results indicate that the blood glucose is elevated. Which prescription would the nurse anticipate for the client?

A 3-hour glucose tolerance test2A sliding-scale regular insulin dose3Administration of an oral hypoglycemic agent4Administration of NPH insulin on a daily basis 1 A maternal glucose is prescribed to screen for gestational diabetes. If it is elevated, a 3-hour glucose tolerance test is recommended to determine the presence of gestational diabetes. A sliding-scale regular insulin dose, an oral hypoglycemic agent, or NPH insulin should not be prescribed based solely on the maternal glucose levels. Further follow-up should be implemented.

While assisting with the measurement of fundal height, the client at 36 weeks' gestation states that she is feeling lightheaded. On the basis of the nurse's knowledge of pregnancy, the nurse determines that this is most likely a result of which reason?

A full bladder2Emotional instability3Insufficient iron intake4Compression of the vena cava 4 Compression of the inferior vena cava and aorta by the uterus may cause supine hypotension syndrome during pregnancy. Having the woman turn onto her left side or elevating the right buttock during fundal height measurement will prevent or correct the problem. Options 1, 2, and 3 are not the cause of the problem described in the question.

A prenatal client diagnosed with anemia has come to the clinic. After reviewing the client's health record, the nurse notes that the laboratory values indicate low hemoglobin and hematocrit levels. Which problem do the data best support?

Anxiety2Low self-esteem3High risk for infection4Cardiovascular accident (stroke) 3 Women with anemia have a higher incidence of puerperal complications such as infection than do pregnant women with normal hematological values. No data in the question support anxiety, low self-esteem, or cardiovascular accident.

The nurse is reinforcing instructions to a client about preterm labor. Which method of teaching would the nurse use?

Ask about contractions at each visit.2Provide a simple pamphlet with multiple illustrations.3Palpate for uterine contractions at the same time as the client.4Attach the monitor to the client's abdomen and have her palpate at the same time. 1,3 Palpating simultaneously with the client uses teaching and learning principles. It includes the most direct way to determine the level of client understanding. The client may not be able to read well. The client may not understand what to feel for with contractions and may answer only to please the nurse. A monitor would be cost prohibitive and does not give human feedback.

The nurse is reviewing the health record of a pregnant client at 16 weeks' gestation. The nurse would expect to document that the fundus of the uterus is located at which area?

At the umbilicus2Just above the symphysis pubis3At the level of the xiphoid process4Midway between the symphysis pubis and the umbilicus 4 At 12 weeks' gestation, the uterus extends out of the maternal pelvis and can be palpated above the symphysis pubis. At 16 weeks, the fundus reaches midway between the symphysis pubis and the umbilicus. At 20 weeks, the fundus is located at the umbilicus. By 36 weeks, the fundus reaches its highest level at the xiphoid process.

A woman who is 8 weeks pregnant complains to the nurse about nausea. Which advice would the nurse provide to this client about ways to assist with this problem? Select all that apply.

Avoid greasy foods.2Eat 5 to 6 small meals each day.3Do not drink fluids with meals.4Eat highly spiced foods only in evening hours.5Refrain from eating anything for 2 to 3 hours after arising. 1,2,3 Nausea is a common problem in the first trimester. The woman is encouraged to avoid fried, greasy, or spicy foods and foods with strong odors, such as cabbage and onions; eat small, frequent meals; eat dry toast or crackers before getting out of bed in the morning; and drink fluids between meals instead of with meals.

A nurse is reinforcing instructions to a client in the first trimester of pregnancy about measures to help with morning sickness. Which would the nurse include in the instructions? Select all that apply.

Avoid milk.2Eat a low-fat diet.3Stop or decrease smoking.4Eat smaller, more frequent meals.5Consume adequate fluid between meals. 2,3,4,5 Some measures for decreasing morning nausea are decreasing fats in the diet; stopping or decreasing smoking; eating smaller, more frequent meals; and consuming adequate fluid between meals, but not with meals. Milk should not be avoided, and milk does not make morning sickness worse.

The nurse is checking a client's record for probable signs of pregnancy. Which are the probable signs of pregnancy that the nurse would note? Select all that apply.

Ballottement2Chadwick's sign3Uterine enlargement4Braxton Hicks contractions5Outline of fetus via radiography or ultrasound6Fetal heart rate detected by a nonelectronic device 1,2,3,4, The probable signs of pregnancy include uterine enlargement; Hegar's sign (the compressibility and softening of the lower uterine segment that occurs at about week 6); Goodell's sign (the softening of the cervix that occurs at the beginning of the second month of pregnancy); Chadwick's sign (the violet coloration of the mucous membranes of the cervix, vagina, and vulva that occurs at about week 4); ballottement (the rebounding of the fetus against the examiner's fingers on palpation); Braxton Hicks contractions; and a positive pregnancy test that measures for human chorionic gonadotropin. Positive signs of pregnancy include a fetal heart rate that is detected by an electronic device (Doppler transducer) at 10 to 12 weeks' gestation and by a nonelectronic device (fetoscope) at 20 weeks' gestation; active fetal movements that are palpable by the examiner; and an outline of the fetus via radiography or ultrasound.

A pregnant client asks the nurse in the clinic when she will be able to start feeling the fetus move. The nurse responds by telling the mother that fetal movements should be noted at which time interval?

Between 4 and 8 weeks' gestation2Between 6 and 10 weeks' gestation3Between 10 and 14 weeks' gestation4Between 16 and 20 weeks' gestation 4 Fetal movement, called quickening, is not perceived until the second trimester. Between 16 and 20 weeks of gestation the expectant mother first notices subtle fetal movements that gradually increase in intensity.

A client in her 24th week of pregnancy is admitted to the hospital in preterm labor. She asks the nurse if her baby will live if the labor cannot be stopped. Which diagnostic test would the nurse expect the primary health care provider to prescribe?

Biophysical profile2Chorionic villus sampling3Ultrasound for amniotic fluid volume4Amniocentesis for fetal surfactant level 4 An amniocentesis is performed to obtain a specimen of fluid to detect the surfactant level. By 20 weeks of gestation the lungs have matured functionally enough for the fetus to survive outside the uterus (age of viability), but special care in the neonatal intensive care unit (NICU) would be required. A biophysical profile consists of a group of five fetal assessments: fetal heart rate and reactivity (the NST), fetal breathing movements, fetal body movements, fetal tone (closure of the hand), and the volume of amniotic fluid (AFI). Chorionic villus sampling consists of obtaining a small part of the developing placenta to analyze fetal cells at 10 to 12 weeks of gestation. The ultrasound scan measures the amniotic fluid pockets in all four quadrants surrounding the mother's umbilicus and produces an amniotic fluid index (AFI). From 5 to 19 cm is considered normal.

The nurse is reviewing the health care record of a pregnant client at 16 weeks of gestation. Which assessment findings are most likely present at this time? Select all that apply.

Blood pressure peaks at 140/90.2The fundus is located at the umbilicus.3Fetal heart tones can be heard by Doppler.4Braxton Hicks contractions may be felt by the mother.5The fundus is located midway between the symphysis pubis and the umbilicus. 3,4,5 By 16 weeks gestation, fetal heart tones can be heard by Doppler and Braxton Hicks contractions may be felt by the mother. At this time, the fundus reaches midway between the symphysis pubis and the umbilicus. Blood pressure during pregnancy should not reach 140/90 at any point in the pregnancy. At 20 weeks, the fundus is located at the umbilicus.

A 26-year-old woman comes to the clinic and asks for a pregnancy test because she thinks she might be pregnant. The nurse assesses for which presumptive signs of pregnancy? Select all that apply.

Breast tenderness2Early morning nausea3Positive urine pregnancy test4Fetal heartbeat heard by Doppler5No menstruation for the last 8 weeks 1,2,5 Presumptive signs of pregnancy are those that can't definitely confirm a diagnosis of pregnancy. These would include amenorrhea, breast changes that include tenderness and pigmentation changes, frequency of urination, and nausea. A positive pregnancy test would be a probable sign, and a fetal heartbeat heard by Doppler is a positive sign of pregnancy.

The nurse is preparing to reinforce instructions to a pregnant client about nutrition. The nurse plans to include which instruction in this client's teaching plan?

Calcium is not important until the third trimester.2All mothers are at high risk for nutritional deficiencies.3Iron supplements are not necessary unless the mother has iron deficiency anemia.4The nutritional status of the mother significantly influences fetal growth and development. 4 Poor nutrition during pregnancy can negatively influence fetal growth and development. Although pregnancy poses some nutritional risk for the mother, not all clients are at high risk. Calcium is critical during the third trimester but must be increased from the onset of pregnancy. Intake of dietary iron is usually insufficient for the majority of pregnant women, and iron supplements are prescribed routinely.

A client asks the nurse to describe how her baby is developing at 12 weeks gestation. Which milestones would the nurse identify as present at this time? Select all that apply.

Can hear2Sex recognizable3Blood forming in marrow4Testes descend into scrotum5Kidneys able to secrete urine 2,3,5 At the end of the 12th week, the fetus' sex is recognizable, blood is forming in the bone marrow, and the kidneys are able to secrete urine. The fetus cannot hear until approximately 24 weeks, and the testes do not descend into the scrotum until approximately 30 weeks.

The nurse is assisting with caring for a client who has a placenta previa. The nurse understands that a cervical examination would not be performed on the client primarily because it could have which consequence?

Cause hemorrhage2Initiate premature labor3Rupture the fetal membranes4Increase the chance of infection 1 Because the placenta is implanted low in the uterus, cervical examination could cause the disruption of the placenta and initiate profound hemorrhage. The other options are also correct, but the hemorrhage is of the greatest concern in this case.

The nurse is reviewing the record of a pregnant client and notes that the primary health care provider has documented the presence of Chadwick's sign. The prenatal client asks the nurse to explain Chadwick's sign. Which information provided by the nurse is accurate? Select all that apply.

Chadwick's sign relates to fundal height.2Chadwick's sign is a probable sign of pregnancy.3Chadwick's sign may be present as early as 6 weeks' gestation.4Chadwick's sign is a bluish discoloration of the vagina and cervix.5Chadwick's sign occurs when the pregnant client experiences fetal movement. 2,3,4 The cervix undergoes significant changes following conception. The most obvious changes occur in color and consistency. Chadwick's sign is a probable sign of pregnancy and may occur as early as 6 weeks' gestation. In response to the increasing levels of estrogen, the cervix becomes congested with blood, resulting in the characteristic bluish color that extends to include the vagina, labia, and cervix. This discoloration, referred to as Chadwick's sign, is one of the earliest signs of pregnancy. Chadwick's sign does not relate to fundal height, and initial fetal movement felt by the pregnant woman is known as quickening.

A client in the third trimester of pregnancy visits the clinic for a scheduled prenatal appointment. The client tells the nurse that she frequently has leg cramps, primarily when she is reclining. On the basis of the client's complaint, which would the nurse do first?

Check for pedal edema.2Check the dorsalis pedis pulses.3Check for signs of thrombophlebitis.4Tell the client to apply heat to the affected area when cramps occur. 3 Leg cramps may be a result of compression of the nerves supplying the legs because of the enlarging uterus, a reduced level of diffusible serum calcium, an increase in serum phosphorus, or the presence of thrombophlebitis. In the pregnant client who complains of leg cramps, the nurse should first check for signs of thrombophlebitis and notify the registered nurse. If thrombophlebitis is not present, the nurse may be instructed to place heat on the affected area, dorsiflex the foot until the spasm relaxes, or have the client stand on a cold surface. The primary health care provider may prescribe oral supplementation with calcium carbonate tablets or calcium hydroxide gel with each meal to increase the calcium level and lower the phosphorus level. Although the nurse may check for edema and check the pedal pulses, these should not be the first actions.

The nurse is reinforcing dietary instructions to a pregnant client with a history of lactose intolerance. The nurse would instruct the client to consume which best food item to ensure an adequate source of calcium in the diet?

Cheese2Spinach3Dried fruits4Orange juice 3 The best source of calcium is dairy products. Women with lactose intolerance need other sources of calcium because they are not able to consume dairy products. Calcium is present in dark green, leafy vegetables; broccoli; legumes; nuts; and dried fruits. Cheese is a dairy product and cannot be eaten by a client who has lactose intolerance. Spinach contains calcium, but it also contains oxalates that decrease calcium availability. Orange juice does not contain significant amounts of calcium unless fortified with calcium.

The nurse is explaining physiological changes of pregnancy that are related to melanocyte-stimulating hormone (melanotropin). Which pregnancy changes are related to the effects of this hormone? Select all that apply.

Chloasma2Linea nigra3Hegar's sign4Darkening of areola5Positive pregnancy test 1,2,4 Melanocyte-stimulating hormone is responsible for the pigment changes of pregnancy. Chloasma, linea nigra, and darkening of the areolas are all integumentary pigment changes. Hegar's sign is the compressibility and softening of the lower uterine segment. A positive pregnancy test is related to the presence of the hormone human chorionic gonadotropin (hCG).

A client is admitted to the hospital and is in the first stage of labor. She tells you that her "bag of waters" broke. Which assessments of the amniotic fluid are considered to be normal? Select all that apply.

Clear fluid2Negative fern test3Foul odor of the amniotic fluid4White flecks in the amniotic fluid5Presence of glucose and protein in the amniotic fluid 1,4,5 The amniotic fluid is clear with a mild odor. White flecks in the amniotic fluid are from lanugo or vernix caseosa. A positive fern test indicates it is amniotic fluid. Amniotic fluid also contains glucose, proteins, and urea. A foul odor indicates the presence of infection

A nursing student is assigned to a client in labor. The nursing instructor asks the student to describe fetal circulation, specifically the ductus venosus. The instructor determines that the student understands the structure of the ductus venosus if the student states which about the ductus venosus?

Connects the pulmonary artery to the aorta2Is an opening between the right and left atria3Connects the umbilical vein to the inferior vena cava4Connects the umbilical artery to the inferior vena cava 3 The ductus venosus connects the umbilical vein to the inferior vena cava. The foramen ovale is a temporary opening between the right and left atria. The ductus arteriosus joins the aorta and the pulmonary artery.

A pregnant client is seen in the health care clinic for a regular prenatal visit. The client tells the nurse that she is experiencing irregular contractions. The nurse determines that the client is experiencing Braxton Hicks contractions. Based on this finding, which nursing action is appropriate?

Contact the primary health care provider.2Instruct the client to maintain bed rest for the remainder of the pregnancy.3Tell the client that these are common and they may occur throughout the pregnancy.4Call the maternity unit and inform them that the client will be admitted in a prelabor condition. 3 Braxton Hicks contractions are irregular, painless contractions that may occur intermittently throughout pregnancy. Because Braxton Hicks contractions may occur and are normal in some pregnant women during pregnancy, options 1, 2, and 4 are unnecessary and inappropriate actions.

The nurse is assisting in conducting a prepared childbirth class and is instructing pregnant women about the method of effleurage. The nurse instructs the women to perform the procedure by taking which action?

Contracting and then consciously relaxing different muscle groups2Massaging the abdomen during contractions using both hands in a circular motion3Instructing the significant other to stroke or massage a tightened muscle by the use of touch4Contracting an area of the body such as an arm or leg and then concentrating on letting tension go from the rest of the body 2 Effleurage is massage of the abdomen during contractions. Women learn to do effleurage using both hands in a circular motion. Progressive relaxation involves contracting and then consciously releasing different muscle groups. Neuromuscular disassociation helps the woman relax her body even when one group of muscles is strongly contracted. In this procedure, the woman contracts an area such as an arm or leg then concentrates on letting tension go from the rest of her body. Touch relaxation helps the woman learn to loosen taut muscles when she is touched by her partner.

The nurse is preparing a woman with gestational hypertension for discharge and shares with the client directions to follow which instructions? Select all that apply.

Curtail exercise.2Weigh yourself every week.3Measure your blood pressure daily.4Rest frequently by lying on your side.5Call the primary health care provider if you develop dizziness. 1,3,4,5 Home care instruction for clients with gestational hypertension include curtailing exercise, avoiding weight loss, measuring blood pressure and weight daily, eating a balanced diet with sufficient protein, reporting problems to the primary health care provider, and encouraging side lying during rest periods.

The client asks the nurse about the purpose of the placenta. The nurse plans to respond to the client knowing which about the placenta?

Cushions and protects the fetus2Maintains the body temperature of the fetus3Surrounds the fetus and allows for fetal movement4Provides an exchange of nutrients and waste products between the mother and the fetus 4 The placenta provides an exchange of nutrients and waste products between the mother and the fetus. The amniotic fluid surrounds, cushions, and protects the fetus and allows for fetal movement. The amniotic fluid also maintains the body temperature of the fetus.

The nurse is assigned to care for a pregnant client with a diagnosis of sickle cell anemia. The nurse plans care, knowing that which problem would receive highest priority?

Dehydration2Inability to perform activities3Verbalizing fear about delivery4Expressing concern about appearance 1 For the client with sickle cell anemia, dehydration will precipitate sickling of the red blood cells. Sickling can lead to life-threatening consequences for the pregnant woman and the fetus, such as an interruption of blood flow to the respiratory system and placenta. Although options 2, 3, and 4 may be components of the plan of care at some point, fluid volume deficit is the priority.

A woman diagnosed previously with gestational hypertension is returning to the clinic for her scheduled prenatal appointment. During the assessment, the nurse is concerned that she is developing signs/symptoms that indicate that her mild gestational hypertension is progressing. What assessment findings indicate to the nurse that the mild gestational hypertension is progressing? Select all that apply.

Denial of visual problems2Braxton Hicks contractions3Negative protein on dipstick of urine4Blood pressure (BP) 165/120 mm Hg5Complaints of headache for the last 12 hours 4,5 Severe gestational hypertension or preeclampsia may be forms of progression of mild gestational hypertension. In a worsening case, the blood pressure (BP) increases above 140/90 as does the proteinuria. The woman begins to have complaints of neurological symptoms. Elevated blood pressure and headaches are correct.

The nurse is working with a pregnant client regarding how to identify the existence of preterm contractions. The nurse plans to use which strategy as an effective teaching method?

Describe the process verbally in great detail.2Palpate for uterine contractions at the same time as the client.3Provide a pamphlet with both multiple pictures and drawings.4Place a monitor on the client's abdomen, and use it as a visual. 2 Palpating contractions with the client is correct because it most fully uses teaching and learning principles. It provides for verification of whether the client can perform the skill and gives immediate feedback about the client's level of understanding. A verbal description may be useful but does not provide for verification that the skill has been learned. Application of a fetal monitor is unnecessary and is more costly and time consuming. Providing written material is incorrect because providing written material does not guarantee that the client has learned the skill.

The nurse is assessing a client during a prenatal visit. The nurse takes the client's temperature and notes that the temperature is 99.2° F. Which nursing action is appropriate?

Document the temperature.2Notify the primary health care provider.3Retake the temperature in 30 minutes.4Inform the client that the temperature is elevated and antibiotics may be required. 1 The normal temperature during pregnancy is 98° to 99.6° F (36.2° to 37.6° C). A temperature above this level suggests infection that might require medical management. Options 2, 3, and 4 are unnecessary.

The pregnant woman complains of being awakened frequently by leg cramps. The nurse reinforces instructions to the client's partner and would tell the client to perform which measure?

Dorsiflex the client's foot while flexing the knee.2Plantarflex the client's foot while flexing the knee.3Dorsiflex the client's foot while extending the knee.4Plantarflex the client's foot while extending the knee. 3 Leg cramps often occur when the pregnant woman stretches her leg and plantar flexes her foot. Dorsiflexion of the foot while extending the knee stretches the gastrocnemius muscle, prevents the muscle from contracting, and halts the cramping. Therefore, the remaining options are incorrect.

A pregnant woman in the second trimester of pregnancy complains of constipation and describes the home care measures she is taking to relieve the problem. Which would the nurse determine is a harmful measure in preventing constipation?

Drinking eight to 10 glasses of water daily2Daily activity such as walking or swimming3Increasing whole grains and fresh vegetables in the diet4Adding 1 tablespoon of mineral oil to a bowl of cereal daily 4 Mineral oil should not be used as a stool softener because it inhibits the absorption of fat-soluble vitamins in the body. Constipation should be treated with increased fluids (six to eight glasses per day) and a diet high in fiber. Increasing exercise is also an excellent way to improve gastric motility.

A perinatal client is at risk for toxoplasmosis. Which instruction would the nurse reinforce with the client to prevent exposure to this disease?

Eat raw meats.2Wash hands only before meals.3Avoid exposure to litter boxes used by cats.4Use topical corticosteroid treatments prophylactically. 3 nfected house cats transmit toxoplasmosis through feces. Handling litter boxes can transmit the disease to the maternity client. Meats that are undercooked can harbor microorganisms that can cause infection. Hands should be washed throughout the day when items that could be contaminated are handled. Topical corticosteroid treatment is not the pharmacological treatment of choice for toxoplasmosis.

A nurse is monitoring a pregnant client for the warning signs/symptoms of gestational hypertension. Which are signs/symptoms of this complication of pregnancy? Select all that apply.

Edema2Polyuria3Proteinuria4Thrombocytopenia5Irregular, painless contractions 1,3,4 Vasospasm that occurs with gestational hypertension impedes blood flow to the mother's organs and placenta, resulting in one or more of these signs: hypertension, edema, and proteinuria (protein in the urine). Severe gestational hypertension can also affect the central nervous system, eyes, urinary tract, liver, gastrointestinal system, and blood clotting function (which results in thrombocytopenia). Oliguria, not polyuria, is a sign. Braxton Hicks contractions are the normal irregular, painless contractions of the uterus that may occur throughout the pregnancy.

A nursing student prepares a teaching plan for a pregnant client newly diagnosed with diabetes mellitus. The nursing instructor suggests changing the plan if the student includes which information?

Effects of diabetes on the pregnancy and fetus2Nutritional requirements for pregnancy and diabetic control3To avoid exercise because of the negative effects on insulin production4To be aware of any infections and report signs of infection immediately to the primary health care provider 3 Exercise is necessary for a pregnant diabetic woman. It is inaccurate to tell a pregnant woman newly diagnosed with diabetes mellitus to avoid exercise. Options 1, 2, and 4 are important points to include in the teaching plan for the pregnant woman recently diagnosed with diabetes mellitus. Concepts related to the timing of exercise, control of food intake, and insulin administration around the time of exercise should be included in the plan.

The nurse is reinforcing instructions to a pregnant client regarding measures to prevent heartburn. The nurse would instruct the client to take which best measure?

Eliminate between-meal snacks.2Drink decaffeinated coffee and tea.3Lie down for 30 minutes after eating.4Substitute salt in cooking for other spices. 2 Caffeine, like spices, may cause heartburn and needs to be avoided. Spices tend to trigger heartburn. Eating smaller, more frequent portions is preferable to eating three large meals to control heartburn. Lying down after meals is likely to lead to the reflux of stomach contents and cause heartburn. Salt leads to the retention of fluid.

The nurse is caring for a client experiencing a partial placental abruption. The client is uncooperative and is refusing any interventions until her husband arrives at the hospital. The nurse analyzes the client's behavior as likely the result of which situation?

Emotional immaturity2A stubborn personality3An undiagnosed psychiatric disorder4Acute anxiety and the need for support 4 Any of the situations identified in the options may contribute to the reason for the client's behavior, but the most likely reason is anxiety. Option 4 is the only option that supports the information identified in the question. The client may be anxious about the unknown effects of complications, and the presence of a support person while dealing with a crisis is crucial. There are no data in the question to support options 1, 2, and 3.

A pregnant client has just been admitted to the hospital with severe preeclampsia. The nurse knows it is important to monitor for additional complications at this time. Which assessment would be part of the plan of care?

Enlargement of the breasts2Complaints of feeling hot when the room is cool3Periods of fetal movement followed by quiet periods4Any bleeding, such as in the gums, petechiae, and purpura 4 Bleeding is an early sign of disseminated intravascular coagulation (DIC), a complication of preeclampsia, and should be reported. Breast enlargement, feeling hot, and having alternate periods of fetal rest and activity are normal occurrences in the last trimester of pregnancy.

The nurse is teaching a pregnant client about the physiological effects and hormone changes that occur in pregnancy. The client asks the nurse about the purpose of estrogen. Which description explains the purpose of estrogen?

Estrogen maintains the uterine lining for implantation.2Estrogen stimulates metabolism of glucose and converts the glucose to fat.3Estrogen prevents the involution of the corpus luteum and maintains the production of progesterone until the placenta is formed.4Estrogen stimulates uterine development to provide an environment for the fetus and stimulates the breasts to prepare for lactation. 4 Estrogen stimulates uterine development to provide an environment for the fetus and stimulates the breasts to prepare for lactation. Progesterone maintains the uterine lining for implantation and relaxes all smooth muscle. Human placental lactogen stimulates the metabolism of glucose and converts the glucose to fat and is antagonistic to insulin. Human chorionic gonadotropin prevents involution of the corpus luteum and maintains the production of progesterone until the placenta is formed.

The nurse is reinforcing instructions to a maternity client on how to keep a fetal activity diary. Which instruction would the nurse provide the client?

Expect the baby to move at least 35 times in 3 hours.2Lie on the stomach when preparing to count the fetal movement.3Schedule the counting periods in the morning when the fetal movement is highest.4Contact the primary health care provider if the baby's movements are fewer than 10 times in 2 hours. 4 Most healthy fetuses move at least 10 times in 2 hours. Slowing or stopping of fetal movement may be an indication that the fetus needs some attention and evaluation. In general, women are advised to count fetal movements for 30 minutes three times a day. The woman should lie on her left side during the procedure because it provides optimal circulation to the uterus-placenta-fetus unit. The time of day may affect fetal movement, which is lower in the morning and higher in the evening.

The nurse is assisting a client who, at 38 weeks of gestation, reports feeling dizzy, lightheaded, and nauseated when attempting to lie down on the examining table. Her skin is pale and is both cool and moist to the touch. Which action would the nurse perform first?

Explain the reason for these symptoms.2Place a cool washcloth on the client's forehead.3Measure blood pressure, pulse, and respirations.4Place a wedge pillow under the client's right side. 4 The symptoms suggest supine hypotension caused by compression of the aorta and inferior vena cava by the gravid uterus. Compression of these vessels is relieved by the placement of a wedge pillow under the woman's right side. Although the actions in the other options may be implemented, they will not eliminate the problem and thus are not performed first.

The nurse is preparing to teach a pregnant client about the warning signs in pregnancy and prepares a list of the warning signs that indicate the need to notify the primary health care provider. Which warning signs would the nurse place on the list? Select all that apply.

Facial edema2Rapid weight gain3Visual disturbances4Generalized edema5Nausea on arising in the morning6The presence of irregular painless contractions 1,2,3,4 Facial or generalized edema, rapid weight gain, and visual disturbances are warning signs in pregnancy. Nausea upon arising is a common discomfort of pregnancy. Braxton Hicks contractions are the normal, regular, painless contractions of the uterus that may occur throughout the pregnancy. Additional warning signs in pregnancy include vaginal bleeding, premature rupture of the membranes, preterm uterine contractions that are normal and regular, change in or absence of fetal activity, severe headache, epigastric pain, persistent vomiting, abdominal pain, and signs of infection.

The nurse is reinforcing instructions to a pregnant client about the warning signs in pregnancy that require the need to notify the primary health care provider. The nurse determines that further teaching is needed if the client states that it is necessary to call the primary health care provider if which occurs?

Facial edema2Rapid weight gain3Visual disturbances4Irregular, painless contractions 4 Clients should be educated regarding the danger signs of pregnancy. Generalized or facial edema, rapid weight gain and visual disturbances are warning signs in pregnancy. Braxton Hicks contractions are the normal, irregular, painless contractions of the uterus that may occur throughout the pregnancy. Additional warning signs in pregnancy include vaginal bleeding, premature rupture of the membranes, preterm uterine contractions that are normal and regular, change in or absence of fetal activity, severe headache, epigastric pain, persistent vomiting, abdominal pain, and signs of infection.

The nurse is monitoring a client who is receiving oxytocin to induce labor. Which assessment findings would cause the nurse to immediately discontinue the oxytocin infusion? Select all that apply.

Fatigue2Drowsiness3Uterine hyperstimulation4Late decelerations of the fetal heart rate5Early decelerations of the fetal heart rate 3,4 Oxytocin stimulates uterine contractions and is a common pharmacological method to induce labor. High-dose protocols have been associated with more uterine hyperstimulation and more cesarean births related to fetal stress. Late decelerations, a nonreassuring fetal heart rate pattern, is an ominous sign indicating fetal distress. Some primary health care providers prescribe the administration of oxytocin in 10-minute pulsed infusions rather than as a continuous infusion. This pulsed method, which is more like endogenous secretion of oxytocin, is reported to be effective for labor induction and requires significantly less oxytocin use. Oxytocin infusion must be stopped when any signs of uterine hyperstimulation are present. Drowsiness and fatigue may be caused by the labor experience. Early decelerations of the fetal heart rate are a reassuring sign and do not indicate fetal distress.

The nursing instructor has taught a lecture on the reproductive cycle of the female and asks a nursing student to identify the functions of the vagina. The student correctly responds by identifying which functions? Select all that apply.

Female organ of coitus2Discharge of menstrual flow3Allows for fetal passage during the process of birth4Assists in propelling the ovum through the fallopian tube5Produces sex hormones that assist in maintaining the pregnancy 1,2,3 The pelvis is a bony structure that supports and protects the lower abdominal and internal reproductive organs. The vagina is the female organ of coitus, allows discharge of the menstrual flow, and assists in the passage of the fetus from the uterus to outside the mother's body during childbirth. The fallopian tubes are lined with folded epithelium containing cilia that beat rhythmically toward the uterine cavity to propel the ovum through the tube. The functions of the ovaries include sex hormone production and maturation of an ovum during each reproductive cycle.

The nurse is collecting data from a client during the first prenatal visit at 12 weeks' gestation. The client is anxious to know what the fetus will look like at this time. The nurse correctly responds to the client by providing which information? Select all that apply.

Fetus is able to hear (24 weeks).2Earliest taste buds present.3Kidneys able to secrete urine.4Lecithin begins to appear in amniotic fluid (weeks 27-28).5Sex can be determined as internal and external organs are sex specific. 2,3,5 At 12 weeks' gestation, the fetus is approximately 6 to 9 cm in length and 19 grams. By the end of the twelfth week, taste buds are present, kidneys are able to secrete urine, and the external genitalia of the fetus have developed to such a degree that the gender of the fetus can be determined visually. The fetus is not able to hear until approximately 24 weeks' gestation. Lecithin begins to appear in the amniotic fluid at approximately 27 to 28 weeks' gestation.

The nurse is assessing a client who is at 32 weeks of gestation. It has been 4 weeks since her last visit. Which assessment needs to be reported to the primary health care provider?

Fundal height, 38 cm2Weight gain, 3 pounds3Blood pressure, 118/70 mm Hg4Fetal heart tones, 144 beats/minute 1 From 22 weeks until term, the fundal height measured in centimeters is roughly plus or minus 2 cm of the gestational age of the fetus in weeks. If the fundal height exceeds weeks of gestation, additional assessment is necessary to investigate the cause for the unexpected uterine size. If an unexpected increase in uterine size is present, it may be that the estimated date of delivery is incorrect, and the pregnancy is more advanced than previously thought. If the estimated date of delivery is correct, it may be possible that more than one fetus is present or excessive amniotic fluid is present. Expected weight gain is a pound a week in the second and third trimesters. The blood pressure and fetal heart tones are in normal ranges.

The nurse is measuring the fundal height of a client who is at 30 weeks of gestation. In preparing to perform the procedure, the nurse would take which action?

Have the client stand for the procedure.2Assist the client from a sitting to a right lateral position.3Place the client in a prone position with the head of the bed elevated.4Place the client in a supine position and place a wedge under the right hip. 4 When measuring fundal height, the client lies in a supine position, and the nurse places a wedge under the right hip. This position will assist in preventing supine hypotension. Standing, right lateral, or prone positions are incorrect client positions for measuring fundal heigh

A client in the prenatal clinic presents with a blood pressure reading of 140/90 mm Hg, which is an elevation from last month's reading of 114/66 mm Hg. Which additional sign or symptom suggests to the nurse that the client has mild preeclampsia?

Headaches2Generalized edema3Weight gain of 10 pounds4Trace amount of protein 4 Preeclampsia is considered mild when the diastolic blood pressure does not exceed 100 mm Hg and proteinuria is no more than 500 mg/day (trace to 1+). Symptoms such as headache, visual disturbances, or abdominal pain are typically absent with mild preeclampsia. Therefore, the only sign of mild preeclampsia from the options given is a trace amount of protein. A rapid weight gain and generalized edema may occur. Headaches are present in severe preeclampsia.

The nurse assists a pregnant client with cardiac disease in identifying resources to help her care for her 18-month-old child during the last trimester of pregnancy. The nurse encourages the pregnant client to use these resources primarily to accomplish which tasks?

Help the mother prepare for labor and delivery.2Reduce excessive maternal stress and fatigue.3Avoid exposure to potential pathogens and resulting infections.4Prepare the 18-month-old child for maternal separation during hospitalization. 2 A variety of factors can cause increased emotional stress during pregnancy, resulting in further cardiac complications. The client with known cardiac disease is at greater risk for such complications. Use of appropriate resources will help the client avoid emotional stress, thus reducing additional cardiac compromise during the last trimester. Helping to prepare for the delivery process, avoiding pathogens and resulting infections and preparing a toddler for maternal separation are not primary purposes for use of resources with the pregnant cardiac client.

A pregnant client tells the nurse that she has been craving "unusual foods." On further data collection, the nurse discovers that the client has been ingesting daily amounts of white clay dirt from her backyard. Which laboratory result indicates a physiological consequence of a result of this practice?

Hematocrit 37% 2Glucose 86 mg/dL 3Hemoglobin 9.1 g/dL 4White blood cell count 12,400/mm3 3 Pica cravings often lead to iron deficiency anemia, resulting in a lowered hemoglobin. The other three laboratory values are within normal limits for the pregnant woman.

The nurse is assisting in developing a teaching plan for a pregnant client diagnosed with diabetes mellitus. Which instruction is the priority for this client?

How to test for proteinuria2How to manage the discomfort of early labor3How to check for and manage preterm bleeding4How to check for signs of hypoglycemia and the required treatment 4 In diabetes mellitus, the pancreas does not produce enough insulin for necessary carbohydrate metabolism. The physiological changes of pregnancy drastically alter insulin requirements. Pregnant clients with diabetes mellitus should be taught to monitor themselves for hypoglycemia to minimize potential maternal and fetal effects that result from hypoglycemia. Testing for proteinuria is important for the mother with gestational hypertension or preeclampsia. Management of preterm bleeding is taught to the mother with placenta previa. Managing the discomforts of early labor is important for all pregnant women.

The nurse is assigned to care for a pregnant client being admitted to the nursing unit. Laboratory and diagnostic studies have confirmed a diagnosis of gestational trophoblastic disease (hydatidiform mole). The nurse collects data on the client and reviews the results of the laboratory and diagnostic studies, knowing that which findings are associated with this diagnosis? Select all that apply.

Hypotension2Vaginal bleeding3Excessive vomiting4No fetal heart activity5Larger than normal uterine size6Elevated levels of human chorionic gonadotropin (hCG) 2,3,4,5,6 Gestational trophoblastic disease is a group of rare tumors that involve abnormal growth of cells inside a woman's uterus. The most common signs and symptoms of gestational trophoblastic disease include elevated levels of hCG, vaginal bleeding, larger than normal uterus, failure to detect fetal heart activity even with sensitive instruments, excessive nausea and vomiting, and early development of gestational hypertension. An elevated blood pressure also should be noted.

The client at 28 weeks' gestation is Rh negative and Coombs antibody negative. The nurse determines that the client understands what the nurse has taught her about Rh sensitization when the client makes which statement?

I know I can never have another child."2"I am glad I won't have to have these shots if I have another child."3"I will have to have an injection once a month until the baby is born."4"I will tell the nurse at the hospital that I had an Rh shot during pregnancy." 4 As described in the question, it is accepted practice to administer Rho(D) immune globulin to an Rh-negative woman at 28 weeks' gestation, with a second injection within 72 hours of delivery. This prevents sensitization, which could jeopardize a future pregnancy. For subsequent pregnancies or abortions, the injections must be repeated, because the immunity is passive. Options 1, 2, and 3 are inaccurate information.

The nursing student is conducting a clinical conference regarding the hormones that are related to pregnancy, and the instructor asks the student about the function of progesterone. Which response made by the student indicates an understanding of the function of this hormone? Select all that apply.

It is the primary hormone of milk production."2"It maintains the uterine lining for implantation."3"It softens the muscles and joints of the pelvis."4"It relaxes all smooth muscle, including the uterus."5"It increases during pregnancy to stimulate the basal metabolic rate." 2,4 Progesterone decreases the mother's ability to utilize insulin. Progesterone maintains the uterine lining for implantation and relaxes all smooth muscle, including the uterus. Prolactin is the primary hormone of milk production. Relaxin is the hormone that softens the muscles and joints of the pelvis during labor. Thyroxine increases during pregnancy to stimulate basal metabolic rate.

The nurse is reinforcing teaching to a pregnant woman about the physiological effects and hormonal changes that occur during pregnancy. The woman asks the nurse about the purpose of estrogen. The nurse bases the response on which purpose of estrogen?

It maintains the uterine lining for implantation.2It stimulates the metabolism of glucose and converts glucose to fat.3It stimulates uterine development to provide an environment for the fetus and stimulates the breasts to prepare for lactation.4It prevents the involution of the corpus luteum and maintains the production of progesterone until the placenta is formed. 3 Estrogen stimulates uterine development to provide an environment for the fetus and it stimulates the breasts to prepare for lactation. Progesterone maintains the uterine lining for implantation and relaxes all smooth muscle. Human placental lactogen stimulates the metabolism of glucose and converts the glucose to fat. Human chorionic gonadotropin prevents the involution of the corpus luteum and maintains the production of progesterone until the placenta is formed.

A nursing student is conducting a clinical conference regarding the hormones related to pregnancy. The instructor asks the student about the function of thyroxine. Which statements by the student indicate an understanding of this hormone? Select all that appl

It softens the muscles and joints of the pelvis."2"It is the primary hormone of milk production."3"It maintains the uterine lining for implantation."4"It may play a role in the neural development of the fetus."5"It increases during pregnancy to stimulate basal metabolic rate 4,5 Thyroxine may lead to a mild enlargement of the thyroid gland while still allowing the pregnant woman to remain euthyroid. Thyroxine increases during pregnancy to stimulate basal metabolic rate. It may also function to assist in the neural development of the fetus. Relaxin is the hormone that softens the muscles and joints of the pelvis. Prolactin is the primary hormone of milk production. Progesterone maintains uterine lining for implantation and relaxes all smooth muscle including the uterus.

A pregnant client in the prenatal clinic states that her last menstrual period (LMP) began April 5 and ended April 12. According to Nägele's rule, which is the estimated date of delivery (EDD)?

January 2. 12January 12 3. January 19 4. December 19 2Nägele's's rule is a noninvasive method of calculating the EDD as follows: subtract 3 months, add 7 days to the first day of the LMP, and add 1 year as appropriate. This is based on the assumption that the cycle is 28 days. April 5 plus 7 days minus 3 months is January 12.

The client arrives at the prenatal clinic for her first prenatal assessment. The client tells the nurse that the first day of her last menstrual period (LMP) was October 20, 2022. Using Nägele's rule, the nurse determines the estimated date of birth is which date?

July 12, 2023 2July 27, 2023 3August 12, 2023 4August 27, 2023 2 The accurate use of Nägele's rule requires that the woman have a regular 28-day menstrual cycle. Subtract 3 months from the first day of the last menstrual period, add 7 days, and then adjust the year as appropriate. In this case, the first day of the LMP was October 20, 2022. When you subtract 3 months, you get July 20, 2022. If you add 7 days, you get July 27, 2022. Add 1 year to this, and you get the estimated date of birth: July 27, 2023.

The nurse is collecting data from a pregnant client with a history of cardiac disease and is checking the client for venous congestion. The nurse inspects which body areas, knowing that venous congestion is commonly noted in which areas? Select all that apply.

Legs2Vulva3Fingers4Around the eyes5Around the abdomen 1,2 Assessment of the cardiovascular system includes observation for venous congestion that can develop into varicosities. Venous congestion is most commonly noted in the legs, vulva, or rectum. Although edema may be noted in the fingers and around the eyes, edema in these areas would not be directly associated with venous congestion. It would be difficult to assess for edema in the abdominal area of a client who is pregnant.

The nurse is reinforcing instructions to a pregnant client regarding the need to consume folic acid in the diet. The nurse determines that the client understands the instructions when the client states that it is necessary to include which food item in the diet?

Rice2Cheese3Chicken4Green, leafy vegetables 4 Sources of folic acid include green, leafy vegetables; whole grains; fruits; liver; dried peas; and beans. Rice, cheese, and chicken are not sources of folic acid.

The client is in her second trimester of pregnancy. She complains of frequent low back pain and ankle edema at the end of the day. The nurse would recommend which measure to help relieve both discomforts?

Lie on the left side with the feet dorsiflexed.2Soak the feet in hot water after performing 10 pelvic tilt exercises.3Lie on the right side with the feet elevated on a pillow and a heating pad on the back.4Lie on the floor with the legs elevated onto a couch or padded chair, with the hips and knees at a right angle. 4 The position described in option 4 will produce the posture of the pelvic tilt while countering gravity as the force that leads to the edema of the lower extremities. Although the other options may seem useful, options 2 and 3 identify heat, which should be prescribed by the health care provider (HCP). Option 1 will not relieve back pain and ankle edema.

The nurse is collecting data from the client about the presence of presumptive, probable, and positive signs of pregnancy. Which are the positive signs of pregnancy? Select all that apply.

Lightening2Quickening3Fetal heart tones4Urinary frequency5Positive pregnancy test6Fetal movements felt by examiner 3,6 Presumptive signs of pregnancy include urinary frequency and quickening. Probable signs of pregnancy include Goodell's sign, Chadwick's sign, Hegar's sign, McDonald's sign, abdominal enlargement, Braxton Hicks contractions, striae, and a positive pregnancy test. Fetal movement felt by the examiner, visualization of the fetus (ultrasound) and the presence of fetal heart tones are positive signs of pregnancy.

The nurse is collecting data from a prenatal client. The nurse determines that which situation places the client in the high-risk category for contracting human immunodeficiency virus (HIV)?

Living in an area where HIV infections are minimal2A history of intravenous (IV) drug use in the past year3A history of one sexual partner within the past 10 years4A heterosexual spouse who has had only one sexual partner in the past 10 years 2 HIV is transmitted by intimate sexual contact and by the exchange of body fluids, exposure to infected blood, and the transmission from an infected woman to her fetus. Women who fall into the high-risk category for HIV infection include those with persistent and recurrent sexually transmitted infections or a history of multiple sexual partners and those who use or have used IV drugs. The remaining options are not situations that contribute to contracting HIV infection.

A pregnant client asks the prenatal clinic nurse what the fetal period of development means. Which is correct information about the fetal period?

Longest period of fetal development2First 3 days of fetal development following conception3First 2 weeks of fetal development following conception4Fetal development beginning the third week after conception through the eighth week 1 The fetal period is the longest part of prenatal development. It begins 9 weeks after conception and ends with birth. All major systems are present in their basic form. The pre-embryonic period is the first 2 weeks after conception. Around the fourth day after conception, the fertilized ovum, now called a zygote, enters the uterus. The embryonic period of development extends from the beginning of the third week through the eighth week after conception. Basic structures of all major body organs are completed during the embryonic period.

The nurse is collecting data on a pregnant client and is preparing to take the client's blood pressure. In which position would the nurse place the client?

Lying down2On the left side3On the right side4In a sitting position 4 Because position affects blood pressure in the pregnant woman, the method for obtaining blood pressure should be standardized as much as possible. The blood pressure should be obtained in the sitting position with the arm supported in a horizontal position at heart level. Lying down or positioning on the left or right side is incorrect, and these positions may cause physiological stress that will affect the blood pressure.

A newly pregnant client is asking how to prevent neural-tube birth defects. The nurse reinforces which food choices to include in the diet? Select all that apply.

Milk2Peanuts3Oranges4Broccoli5Egg yolks6Grapefruit 3,4,6 Folic acid (folate) helps prevent neural tube birth defects; it is found in green, leafy vegetables; liver, beef, and fish; legumes; and grapefruit and oranges. Peanuts are high in protein and niacin. Milk is high in calcium and vitamin D. Egg yolks are high in vitamin A, iron, and cholesterol.

The nurse is providing health care information to a pregnant client who is human immunodeficiency virus (HIV) positive. The nurse instructs the client that it is important to avoid alcohol and cigarettes during pregnancy and to get adequate rest primarily to accomplish which goal?

Minimize the possibility of preterm labor.2Reduce the risks of anemia during pregnancy.3Avoid further stress on the maternal immune system.4Minimize the risk of premature rupture of membranes 3 The use of alcohol and cigarettes during the pregnancy of an HIV-infected client, as well as not getting appropriate rest, can compromise the maternal immune system and interfere with medical treatments that may be in place. Collectively, such factors may place both the mother and fetus at additional risk during the pregnancy. Option 3 identifies the primary nursing management subject for the HIV-infected client.

The nurse encourages the childbearing woman diagnosed with human immunodeficiency virus (HIV) to avoid alcohol and cigarettes during pregnancy and to obtain adequate rest. Which outcome is specific to this client?

Minimize the possibility of preterm labor.2Reduce the risks of anemia during pregnancy.3Minimize the potential for developing infections.4Minimize the risk of premature rupture of membranes. 3 The pregnant client with HIV needs to avoid practices that can compromise the maternal immune system and interfere with medical treatments that may be in place. Collectively, such practices may place both the mother and fetus at additional risk during the pregnancy. The remaining options are not as specific to the care of this client.

The nurse assigned to care for a client with mild preeclampsia would anticipate which specific nursing intervention for this client?

Monitoring fetal movement2Maintaining complete bed rest3Monitoring daily blood glucose4Restricting maternal fluid intake 1 A client with mild preeclampsia can be managed at home. The expectant mother is asked to keep a record of fetal movements. Bed rest with bathroom privileges is prescribed. Urine is checked for protein. A blood glucose test is not necessary. The client usually follows a regular diet that does not restrict fluids.

The nurse is reading the primary health care provider's documentation regarding a pregnant client and notes that the primary health care provider has documented that the client has an android pelvic shape. Which descriptions apply to an android pelvis? Select all that apply.

Narrow wedge shape2Long, narrow oval shape3Favorable for a vaginal birth4Short, flattened, wide oval shape5Unfavorable for a vaginal birth 1,5 The android pelvis is wedge-shaped and narrow and is unfavorable for a vaginal birth. A gynecoid pelvic shape is rounded with a wide pubic arch and is the most favorable shape for a vaginal birth. An anthropoid pelvis is long, narrow, and oval. It is not as favorable of a shape for a vaginal birth as the gynecoid pelvis; however, it is a more favorable pelvic shape than the platypelloid or android. The platypelloid pelvis is flattened with a wide, short, oval shape and is also an unfavorable shape for a vaginal birth.

The nurse is assisting in planning care to meet the emotional needs of a pregnant woman. Which nursing intervention is least likely to assist in meeting her emotional needs?

Offering praise and reinforcement for compliance with treatment therapies2Using a caring and supportive approach when dealing with a pregnant woman3Providing the mother with pamphlets and booklets to read about the pregnancy4Providing an opportunity for the pregnant woman to discuss the aspects of pregnancy 3 The woman's emotional needs can be met by providing regular opportunities for discussing aspects of her pregnancy and prenatal care, by using a caring and supportive approach, and by offering praise and reinforcement. The nurse also should discuss the emotional changes of pregnancy, family alterations, and changes in marital relationships that may occur. Providing written materials does not provide nurse-client interaction.

The nurse is told that a prenatal client is at risk for placental abruption. The nurse expects to note which risk factor documented in the client's record?

Oliguria2Gestational diabetes3Maternal hypertension4Hyperemesis gravidarum 3 Maternal hypertension is a risk factor associated with placental abruption. This factor leads to degenerative changes in the small arteries that supply the intervillous spaces of the placenta. This results in thrombosis, causing retroplacental hematoma and leading to placental separation. Oliguria, gestational diabetes, and hyperemesis gravidarum are not associated risk factors.

The nurse is collecting data on a client with severe preeclampsia. Which signs and symptoms are noted in severe preeclampsia? Select all that apply.

Oliguria2Seizures3Contractions4Proteinuria 3+5Muscle cramps 1,4,6 Severe preeclampsia is characterized by blood pressure higher than 160/110 mm Hg, proteinuria 3+ or higher, and oliguria. Seizures (convulsions) are present in eclampsia and are not a characteristic of severe preeclampsia, although treatment is aimed at preventing eclampsia, which is characterized by seizures. Muscle cramps and contractions are not findings noted in severe preeclampsia, although the client is monitored for these occurrences.

A hepatitis B screen is performed on a pregnant client, and the results indicate the presence of antigens in the maternal blood. Which does the nurse anticipate to be prescribed?

Repeating hepatitis screen2Retesting the mother in 1 week3Administration of antibiotics during pregnancy4Administration of immune globulin and vaccine in the infant soon after birth 4 A hepatitis B screen is performed to detect the presence of antigens in maternal blood. If antigens are present, the infant should receive immune globulin and a vaccine soon after birth. Options 1, 2, and 3 are incorrect actions or treatment measures.

A pregnant client with mitral valve prolapse is receiving anticoagulant therapy during pregnancy. The nurse collects data on the client and expects which prescription?

Oral intake of 15 mg of warfarin daily2Subcutaneous administration of terbutaline3Intravenous infusion of heparin sodium 5000 units daily4Subcutaneous administration of heparin sodium 5000 units daily 4 Pregnant women with mitral valve prolapse are frequently given anticoagulant therapy during pregnancy because they are at greater risk for thromboembolic disease during the antepartum, intrapartum, and postpartum periods. Warfarin is contraindicated during pregnancy because it crosses the placental barrier, causing potential fetal malformations and hemorrhagic disorders. Heparin sodium, which does not cross the placental barrier, is safe to use during pregnancy and should be administered by the subcutaneous route. Terbutaline is indicated for preterm labor management only.

The nurse is reading the primary health care provider's (PHCP) documentation regarding a pregnant client and notes that the PHCP has documented that the client has an android pelvic shape. The nurse understands that which characteristics are included with this pelvic shape? Select all that apply.

Oval shaped2Heart shaped3Straight sidewalls4Convergent sidewalls5Wide suprapubic arch6Narrow interspinous diameter 2,4,6 A gynecoid pelvic shape is rounded with a wide pubic arch and is the most favorable pelvic shape for a vaginal birth. The android pelvic shape is heart-shaped and narrow and is an unfavorable shape for a vaginal birth. An anthropoid pelvic shape is long, narrow, and oval. It is not as favorable for a vaginal birth as the gynecoid pelvic shape, which has straight sidewalls and a wide suprapubic arch; however, it is a more favorable pelvic shape than the platypelloid or android. The platypelloid pelvic shape is flattened with a wide, short oval shape and is an unfavorable shape for a vaginal birth

The nurse is conducting a prenatal session with a group of expectant parents. The nurse recognizes that teaching regarding hormones has been successful if a parent makes which statement?

Oxytocin assists with the maintenance of pregnancy."2"Prolactin is the hormone responsible for the secretion of milk."3"Progesterone is needed for growth of pubic and axillary hair at puberty."4"Testosterone is produced by the pituitary gland and assists with breast development." 2 Prolactin stimulates the secretion of milk. Oxytocin may assist in the onset of labor and stimulates contractions during birth and stimulates postpartum contractions to compress uterine vessels and control bleeding. Oxytocin is also responsible for the "let-down" reflex associated with lactation, but it is not responsible for maintenance of the pregnancy. Progesterone stimulates the secretions of the endometrial glands, causing endometrial vessels to become highly dilated and tortuous in preparation for possible embryo implantation. Testosterone is produced by the adrenal glands in the female and induces the growth of pubic and axillary hair at puberty.

During the first trimester of pregnancy, a client complains of frequent nausea followed by vomiting. On data collection, which finding indicates a serious nutritional disorder of pregnancy?

Patellar reflex is 2+2Chadwick's sign is positive3Ketone bodies in urine are negative4Weight compared to last visit is a loss of 2.3 pounds 4 Weight loss along with the signs/symptoms described in the question could indicate hyperemesis gravidarum. Patellar reflexes would be evaluated during magnesium sulfate administration. A patellar reflex of 2+ is considered normal. Chadwick's sign may be an indicator of pregnancy. Ketone bodies, if present, would indicate protein wasting.

The nurse is assisting with care for a pregnant client in labor who will be delivering twins. The nurse prepares to monitor the fetal heart rates by performing which?

Placing external fetal monitors so that each fetal heart rate is monitored separately2Placing the external fetal monitor over the fetus that is most anterior to the mother's abdomen3Placing the external fetal monitor over the fetus that is most posterior to the mother's abdomen4Placing the fetal monitor so that one fetus is monitored for a 15-minute period followed by a 15-minute fetal monitoring period for the second fetus 1 In a client with a multifetal pregnancy, each fetal heart rate is monitored separately. Options 2, 3, and 4 are incorrect because these actions would not provide information regarding the status of each fetus.

During initial data collection of a client who is pregnant, the nurse notes that the laboratory report shows leukopenia, thrombocytopenia, anemia, and an elevated erythrocyte sedimentation rate. The nurse suspects human immunodeficiency virus (HIV). Which laboratory study further supports the presence of HIV?

Platelet count2Angiotensin levels 3 T lymphocyte levels4Glomerular filtration rate 3 HIV has a strong affinity for surface marker proteins on lymphocytes. This affinity of HIV for T lymphocytes leads to significant cell destruction. Platelet count is important and may be an indicator of HIV, but this laboratory test already has been identified in the data of the question. Angiotensin is produced in the kidney. Glomerular filtration rate indicates kidney function.

Which history places a maternity client at risk for uterine rupture?

Preterm labor2Placenta previa3Abruptio placentae4Cesarean section birth 4 A client with a history of a previous cesarean birth is at most risk for uterine rupture. When a client has a cesarean delivery, an incision is made in the uterine wall. The site of the incision can produce a weakened area in the uterine wall. The conditions identified in options 1, 2, and 3 do not place the client at risk for uterine rupture.

A pregnant client asks the nurse about the hormone that stimulates postpartum contractions. The nurse tells the client that which primary hormone stimulates postpartum contractions?

Prolactin2Oxytocin3Progesterone4Testosterone 2 Oxytocin stimulates contractions during birth and stimulates postpartum contractions to compress uterine vessels and control bleeding. Prolactin stimulates the secretion of milk, called lactogenesis. Progesterone stimulates the secretions of the endometrial glands and causes the endometrial vessels to become dilated and tortuous in preparation for possible embryo implantation. Testosterone is produced by the adrenal glands in the female and induces the growth of pubic and axillary hair at puberty.

The nurse is preparing to collect data on a client with a possible diagnosis of ectopic pregnancy. Which would the nurse check first?

Pulse2Weight3Temperature4Abdominal girth measurement 1 The primary concern when ectopic pregnancy is suspected is the occurrence of bleeding and hypovolemic shock. Pulse is the only assessment that would provide information related to this occurrence. An elevated pulse is an indicator of shock. The nurse should also monitor for decreasing hematocrit levels and pain. Weight, temperature, and abdominal girth measurement do not provide data that indicate the occurrence of hypovolemic shock.

A primipara is being evaluated in the clinic during her second trimester of pregnancy. Which occurrence indicates an abnormal physical finding that necessitates further testing?

Quickening2Braxton Hicks contractions3Consistent increase in fundal height4Fetal heart rate of 180 beats per minute 4 The fetal heart rate depends on the gestational age. It is 160 to 170 beats per minute during the first trimester, and it slows with fetal growth to approximately 120 to 160 beats per minute. Options 1, 2, and 3 are normal expected findings.

A client who consumes alcohol frequently is in the first trimester of pregnancy. Which is the expected outcome when the nurse initiates interventions to assist the client to cease alcohol consumption?

Reducing the potential for fetal growth restriction in utero2Promoting the normal psychosocial adaptation of the mother to pregnancy3Minimizing the potential for placental abruptions during the intrapartum period4Reducing the risk of teratogenic effects to developing fetal organs, tissues, and structures 4 The first trimester "organogenesis" is characterized by the differentiation and development of fetal organs, systems, and structures. The effects of alcohol on the developing fetus during this crucial period depend not only on the amount of alcohol consumed, but also on the interaction of quantity, frequency, type of alcohol, and other drugs that may be abused during this period by the pregnant woman.

A client is seen in the health care clinic for complaints of vaginal bleeding and mild abdominal cramping. On further data collection, the nurse notes that the client's last menstrual period was 10 weeks ago. The client reports that a home pregnancy test was performed and the results were positive. On physical examination, it is noted that the client has a dilated cervix. The nurse understands that the client is at risk for which type of abortion?

Septic2Inevitable3Incomplete4Threatened 2 An inevitable abortion is a termination of pregnancy that cannot be prevented. Moderate to severe bleeding with mild abdominal cramping and cervical dilation is present. An incomplete abortion presents with heavy bleeding, severe cramping, cervical dilation, and passage of large clots. A threatened abortion presents with slight to moderate bleeding and intermittent cramping, but no dilation. A septic abortion presents with bleeding with odor, cervical dilation, and fever. Cramping may be present.

The nurse is collecting data from a client on her first prenatal visit. Which factor indicates that the client is at risk for developing gestational diabetes during this pregnancy?

She has a history of chronic hypertension.2She is 5 feet, 2 inches tall and weighs 175 pounds.3There is a family history of type 1 diabetes mellitus.4Her previous two babies were delivered by cesarean section. 1 Known risk factors that increase the risk of developing gestational diabetes include obesity (more than 198 lb), chronic hypertension, family history of type 2 diabetes mellitus, previous birth of a large infant (more than 4000 g), and gestational diabetes in a previous pregnancy. Options 2, 3, and 4 are not risk factors.

The nurse is collecting data from a client with placenta previa during an office visit. The nurse would check which item as first priority?

Signs of fetal distress2Availability of support systems3Compliance with activity limitations4Client's understanding of her condition 1 Although all of the options may be assessed, the safety of the mother-infant dyad is the priority. Fetal distress is a primary concern, although the information gained through the other assessments may ultimately affect the well-being of the fetus.

A pregnant client in the second trimester of pregnancy is admitted to the maternity unit with a suspected diagnosis of abruptio placentae. Which finding would the nurse expect to note if this condition is present?

Soft uterus2Abdominal pain3Nontender uterus4Painless vaginal bleeding 2 Classic signs and symptoms of abruptio placentae include vaginal bleeding, abdominal pain, uterine tenderness, and contractions. Mild to severe uterine hypertonicity is present. Pain is mild to severe and either is localized over one region of the uterus or is diffuse over the uterus, with a boardlike abdomen. Painless vaginal bleeding and a soft, nontender uterus in the second or third trimester of pregnancy are signs of placenta previa.

The nurse is reviewing the record of a pregnant client and notes that the primary health care provider has documented the presence of Chadwick's sign. Which clinical finding supports the documentation of Chadwick's sign?

Softening of the cervical tip2Softening of the uterine isthmus3Violet bluish color of vaginal mucosa and cervix4Palpating the floating fetus by bouncing it gently and feeling the rebound 3 The cervix undergoes significant changes following conception. The most obvious changes occur in color and consistency. In response to the increasing levels of estrogen, the cervix becomes congested with blood, resulting in the characteristic bluish tinge that extends to include the vagina and labia. This discoloration, referred to as Chadwick's sign, is one of the earliest signs of pregnancy. Softening of the cervical tip is Goodell's sign. Softening of the uterine isthmus is Hegar's sign. Rebounding of the fetus is known as ballottement.

A pregnant client is seen in the health care clinic with reports of morning sickness. When the client asks the nurse about measures to relieve this situation, what is the nurse's appropriate suggestion?

Switch to a high-carbohydrate diet.2Eat a high-protein snack at bedtime.3Consume dry crackers before getting out of bed.4Increase fluids with both meals and with snacks. 3 Some strategies for decreasing morning sickness are keeping crackers, melba toast, or dry cereal at the bedside to eat before getting up in the morning; eating smaller, more frequent meals; decreasing fats; and consuming adequate fluid between meals.

The nurse is reviewing the health history of a pregnant client. Which data noted in the client's health history would indicate a risk for spontaneous abortion?

Syphilis2Age of 45 years3Diabetes mellitus4Prior history of genital herpes 1 Maternal infections such as syphilis, toxoplasmosis, and rubella are causes of spontaneous abortion. There is inconclusive evidence that genital herpes is a causative agent in abortion. Maternal age older than 40 years and diabetes mellitus are considered high risk factors in a pregnancy, increasing the risk for congenital malformations.

The nurse reinforces instructions to a pregnant client regarding the administration of iron. The nurse determines that the teaching is effective if the client states that she will take the iron with which food items?

Tea2Milk3Water4Tomato juice 4 Foods containing ascorbic acid (vitamin C), such as tomato juice, may increase absorption of iron. Additionally, absorption of iron is affected by many substances. Calcium and phosphorus in milk and tannin in tea decrease iron absorption. Water will not act to increase the absorption of the iron.

Which would be included in the plan of care for a pregnant teenager to reinforce instructions regarding dental care?

Tell the dental office staff that she is pregnant.2Avoid the use of local anesthetics during dental work.3Use toothpaste with baking soda to decrease plaque buildup.4Expect to lose at least one tooth because of calcium and phosphorus leaving the teeth to nourish the fetus. 1 It is important to continue dental care during pregnancy. The dental staff needs to know about the pregnancy so that care is taken during examinations and x-ray studies are avoided. Local anesthetics for minor dental work do not have adverse effects on the fetus. Baking soda may irritate the gums, which are more likely to bleed because of the hormonal changes of pregnancy. Tooth loss during pregnancy is not expected.

The client is undergoing an amniocentesis at 16 weeks' gestation to detect the presence of biochemical or chromosomal abnormalities. Which instructions would the nurse reinforce to the client?

The bladder must be full during the examination.2The bladder must be empty during the examination.3She should not eat or drink anything 4 to 6 hours before the examination.4She will be given Rho(D) immune globulin because she is Rh positive. 1 Before 20 weeks' gestation, the bladder must be kept full during amniocentesis to support the weight of the uterus. After 20 weeks' gestation, the bladder should be emptied to minimize the chance of puncturing the placenta or fetus. Rho(D) immune globulin is administered to Rh-negative women because of the risk of contact with the fetal blood during the examination. There are no fluid or food restrictions. Monitoring the fetal heart tones and the vital signs throughout and after the examination is an important intervention

The nurse is assisting in planning care for a client with a diagnosis of placenta previa. The nurse identifies which as the priority goal for the client?

The client exhibits no signs of fetal distress.2The client expresses an understanding of her condition.3The client identifies and uses available support systems.4The client demonstrates compliance with activity limitations. 1 Option 1 clearly identifies a physiological need. Understanding her condition, utilizing support systems and complying with activity limitations may be components of the plan of care, but the physiological integrity and safety of the mother newborn dyad are the priorities.

When caring for the pregnant client with human immunodeficiency virus (HIV), which goal is appropriate?

The client is assisted with the grief process.2The client is advised of an HIV support group.3The client will not have sexual relations during the remainder of pregnancy.4The client will not develop an opportunistic infection during the remainder of pregnancy. 4 HIV is caused by a retrovirus that infects T lymphocytes. This disables the body's ability to fight infection. Nursing goals are directed at the prevention of infections. Options 1 and 2 are interventions, not goals. Sexual relations are not contraindicated with the proper use of protective devices.

A pregnant client is newly diagnosed as having gestational diabetes. She cries during the interview and keeps repeating, "What have I done to cause this? If I could only live my life over." Which client problem would initially direct nursing care at this time?

The client is blaming herself.2The client is experiencing fetal distress.3The client is concerned about her appearance.4The client lacks knowledge regarding diabetes treatment. 1 The client is putting the blame for the diabetes on herself. She is expressing fear and grief. There are no data in the question that indicate that the client lacks knowledge about diabetes treatment, is concerned about appearance, or is experiencing fetal distress.

A client beginning week 30 of gestation comes to the clinic for a routine visit. Which observation by the nurse indicates a need for further teaching?

The client is wearing pantyhose.2The client is wearing nonslip shoes.3The client is wearing knee-high hose.4The client is wearing shoes with arch supports. 3 Varicose veins often develop in the lower extremities during pregnancy. Any constricting clothing, such as knee-high hose, impedes venous return from the lower legs and thus places the client at higher risk for developing varicosities. Clients should be encouraged to wear support hose (pantyhose). Flat, nonslip shoes with proper arch support are important to help the pregnant woman maintain proper posture and balance and minimize fall risks.

The nurse is assisting in developing goals for the postpartum client who is at risk for infection. Which goal would be appropriate?

The client will no longer have leg pain.2The client will verbalize a reduction of pain.3The client will report that an infection is likely to occur.4The client will be able to identify measures to prevent infection. 4 The uterus is theoretically sterile during pregnancy until the membrane ruptures, after which it is capable of being invaded by pathogens. Puerperal infection is a major cause of maternal morbidity and mortality. Options 1 and 2 are not directly related to infection. Option 3 is inaccurate.

The nurse is collecting data from a pregnant client with a history of cardiac disease. The nurse is checking for venous congestion. The nurse inspects which area, knowing that venous congestion is most commonly noted where?

Vulva2Fingers3Around the eyes4Around the abdomen 1 Assessment of the cardiovascular system includes observation for venous congestion that can develop into varicosities. Venous congestion most commonly is noted in the legs, vulva, or rectum. It would be difficult to assess for edema in the abdominal area of a client who is pregnant. Although edema may be noted in the fingers and around the eyes, edema in these areas would not be associated directly with venous congestion.

The nurse is providing instructions to a pregnant client with genital herpes about the measures that need to be implemented to protect the fetus. Which instruction should the nurse provide to the client?

Total abstinence from sexual intercourse is necessary during the entire pregnancy.2Sitz baths need to be taken every 4 hours while awake if vaginal lesions are present.3A cesarean section will be necessary if vaginal lesions are present at the time of labor.4Daily administration of acyclovir is necessary during the first trimester of the pregnancy. 3 For women with active lesions, either recurrent or primary at the time of labor, delivery should be by cesarean section to prevent the fetus from being in contact with the genital herpes. The safety of acyclovir has not been established during pregnancy and should be used only for a life-threatening infection. Clients should be advised to abstain from sexual contact while the lesions are present. If this is an initial infection, they should continue to abstain until they become culture-negative because prolonged viral shedding may occur in such cases. Keeping the genital area clean and dry will promote healing.

The nurse is monitoring a client with mild gestational hypertension (GH). Which data indicate that GH is a concern?

Urinary output has increased.2There is no evidence of proteinuria.3The client complains of a headache and blurred vision.4The blood pressure reading has returned to the prenatal baseline. 3 Options 1, 2, and 4 are all signs that gestational hypertension is not present. Option 3 is a symptom of the worsening of the gestational hypertension and is a concern that needs to be reported.

The nurse is assisting in teaching a series of classes on maintaining a healthy pregnancy. The goal for the class is "The pregnant woman will verbalize measures that may prevent physical traumatic conditions distressing to the fetus." Based on this goal, which topic would be a part of the teaching plan for this class?

Use of over-the-counter medications2Fetotoxic substances in the workplace3Effects of secondary cigarette smoke on the fetus4Travel precautions and use of shoulder seat belts 4 Placental separation as a result of uterine distortion can occur from trauma, such as in car accidents, and decreases or shuts off uteroplacental circulation. Partial placental separation will also result in fetal distress, with the amount of distress depending on the degree of separation. Complete separation leads to sudden severe fetal distress followed by fetal death. Use of the shoulder seat belt decreases the risk of placental separation by preventing the traumatic flexion of the woman's body from sharp braking or impact, if an accident occurs. Although consuming over-the-counter medications, exposure to fetotoxic substances in the workplace, and secondary effects of smoking are important teaching points, they are not related to physical trauma conditions affecting the fetus.

In the prenatal clinic, the nurse is gathering data from a new client for the health history information. Which action is the best way for the nurse to elicit correct responses to questions that refer to sexually transmitted infections?

Use specific closed-ended questions.2Omit this area of questions because they are highly personal.3Establish a therapeutic relationship between the nurse and pregnant client.4Apologize for the embarrassment that these questions may cause the client. 3 Utilizing therapeutic communication skills is necessary when sensitive information is needed. The initial assessment interview establishes the therapeutic relationship between the nurse and the pregnant woman. It is planned, purposeful communication that focuses on specific content. Using closed-ended questions, omitting assessment of this area, or apologizing for any embarrassment the questioning may cause are incorrect options and would not elicit correct client responses.

The primary health care provider is performing a vaginal examination on a pregnant woman. Which assessments are considered to be normal physiological changes in the vagina? Select all that apply.

Vaginal mucosa thins.2Vaginal secretions increase.3Vaginal pH becomes more alkaline.4Bluish discoloration of the vagina.5Higher levels of glycogen in vaginal secretions. 2,4,5 Vaginal secretions increase and have a higher level of glycogen. Chadwick's sign is a bluish discoloration of the vagina caused by increased vascular congestion. The vaginal mucosa thickens. The vaginal secretions become acidic to prevent infections.


Conjuntos de estudio relacionados

المحاضرة الثانية -مبادئ القانون

View Set

MAKRO 25.- 31. Fejezet- fogalmak

View Set

Chapter 1: A Framework for Financial AccountingAssignment

View Set

Chapter 17/1: Florida General Regulations

View Set

Health Assessment- 9, 10, and 11

View Set